ROSH Women's Health EOR QBANK

Ace your homework & exams now with Quizwiz!

28-year-old woman who is morbidly obese presents to her primary care office with reports of irregular menstrual periods. On physical exam, there is increased hair growth to her upper lip, jawline, and abdomen. Her most recent labs are consistent with an elevated hemoglobin A1C, elevated LH, and elevated testosterone. Which of the following would most likely be found on this patient's pelvic ultrasound? Focal heterogeneous mass Free fluid within the posterior cul-de-sac Thickened endometrial stripe "String of pearls" sign

"String of pearls" sign PCOS US: enlarged ovaries with peripheral cysts

22-year-old G1P0 woman at 38 weeks gestation presents to the emergency department with contractions. On physical examination, the baby's head is at the level of the maternal ischial spines. Which of the following is the correct station? +5 station 0 station −2 station −5 station

0 station

25-year-old G1P0 woman at 11 weeks gestation presents to her obstetrician for routine prenatal care. She has questions about how much calcium she should be taking. She has no significant medical history. She has annual blood work drawn by her primary care provider and has not had any abnormal results. Which of the following is the daily recommendation of calcium for this patient? 1,000 mg 2,000 mg 5,000 mg 500 mg

1,000 mg

24-year-old G2P1 woman at 32 weeks gestation presents to her obstetrician for a nonstress test. Which of the following fetal heart rates is considered normal? 110-160 bpm with absent variability 110-160 bpm with moderate variability 170-220 bpm with absent variability 170-220 bpm with moderate variability

110-160 bpm with moderate variability

2-year-old girl presents to the clinic with her mother. Which of the following is accurate patient education regarding the vaccine that helps to prevent the most common sexually transmitted infection? 12-year-old patients starting the vaccine series require two doses The vaccine can clear current infections The vaccine changes the recommended approach to cervical cancer screening The vaccine has not been proven to reduce the risk of anogenital cancers

12-year-old patients starting the vaccine series require two doses

23-year-old woman at 37 weeks gestation is being monitored in the labor and delivery unit after being admitted for persistent contractions and prelabor rupture of membranes. Her blood pressure is 118/78 mm Hg and her pulse is 88 bpm. A Doppler fetal monitor is used to determine the heart rate of the fetus. Which of the following fetal heart rates is within normal range? 109 bpm 142 bpm 161 bpm 175 bpm

142 bpm normal range: 110-160

12-year-old girl presents with concerns about not having started menstruating. She states that all her friends have, and she is afraid something is wrong with her. On physical exam, you note breast development is appropriate for her age. In the absence of menses, at what age would the patient require an evaluation for primary amenorrhea? 13 14 15 16

15 -- age 15 with *normal* secondary sex characteristics -- age 13 with *absence of secondary sex characteristics

35-year-old woman with a 15 pack-year history presents with postcoital bleeding and dyspareunia. On exam, you note a white, sharply demarcated lesion of the cervix after acetic acid is applied. You suspect cervical intraepithelial neoplasia. Which of the following types of human papillomavirus is the most likely type to be present in this patient? 11 16 18 6

16

24-year-old G1P0 woman presents to the clinic for her first prenatal exam. Based on her last menstrual period, she is 6 weeks pregnant. The patient asks when she will start feeling her baby move. At which of the following gestational ages is this patient most likely to start feeling fetal movements? 15 weeks 19 weeks 21 weeks 23 weeks

19 weeks fetal quickening: primiparous 18-20 weeks, multiparous as early as 14 weeks

22-year-old woman presents to the obstetric clinic for her first prenatal visit at 3.5 weeks gestation. She has a human chorionic gonadotropin of 600 mIU/mL. When would the human chorionic gonadotropin be expected to have doubled in value in a normal pregnancy? 1 day 2 days 4 days 5 days

2 days doubles every 29-53 hours in first 30 days

26-year-old G3P1102 pregnant woman presents to the obstetric clinic for a routine prenatal visit. Physical examination reveals a uterine fundal height near the umbilicus. Which of the following gestational ages is most consistent with this fundal height, assuming a normal pregnancy? 12 weeks 16 weeks 20 weeks 36 weeks

20 weeks 12 - pubic symphysis 20 - umbilicus 36 - xiphoid process

32-year-old G5P4 woman presents for her routine obstetrics visit. She notes concern due to a previous history of gestational diabetes during her last pregnancy. At how many weeks is it appropriate to screen for this condition? 10-12 weeks 15-20 weeks 24-28 weeks 35-37 weeks

24-28 weeks do non fasting 50g oral glucose challenge

During a neonatal assessment at 1 minute, a neonatal boy has a heart rate of 86 bpm and an irregular respiratory rate. He has his upper and lower extremities held in flexion with little movement. His skin is blue around his hands and feet, and he responds to pain with a grimace but no crying. Which of the following is the correct Apgar score? 3 5 7 9

5

24-year-old G1P0 woman presents to her obstetrician after a positive home pregnancy test. She has a history of irregular menses and states that her last menstrual period was 10 weeks ago. A transvaginal ultrasound is ordered. At how many weeks gestation can the yolk sac typically be visualized? 10 weeks 16 weeks 2 weeks 5 weeks

5 weeks on transvaginal US gestational sac: 4.5-5 wks yolk sac: 5-6 wks cardiac activity: 5.5-6 wks

52-year-old woman presents to the clinic with a painless breast mass that she noticed a month ago. Physical exam reveals a breast mass with poorly defined borders that is fixed to the underlying chest wall and palpable left-sided axillary lymphadenopathy. Mammogram reveals an irregular and spiculated lesion. At which of the following ages does the United States Preventive Services Task Force recommend initiating screening for the suspected diagnosis? 40 years 45 years 50 years 55 years

50 years and every 2 years for average-risk

32-year-old G3P2 woman presents to the office complaining of heavy menstrual bleeding. Menarche occurred at 12 years of age, and she reports a history of regular menses every 28 days since. After the birth of her second child, her menses were regular, but she now reports heavy menstrual bleeding. She is able to fill a super maxi pad every 2 hours. Her last Pap smear was performed 6 months ago and was normal. Transvaginal ultrasound reveals an 8 cm uterus with a 6 mm endometrial stripe. There are no uterine fibroids. Ovaries are normal and there are no adnexal masses. She is interested in a low-maintenance treatment that will prevent heavy bleeding or one that would cause menses to cease altogether. She is not interested in endometrial ablation or hysterectomy at this time since she is unsure whether she desires to preserve childbearing potential for the future. Which of the following clinical therapeutics is the most

52 mg levonorgestrel-releasing intrauterine device

24-year-old woman presents to the emergency room with right-sided pelvic pain and vaginal bleeding. Vital signs are T of 98.4°F, HR of 108 bpm, BP of 124/78 mm Hg, RR of 20/min, and oxygen saturation of 99% on room air. The patient has right-sided adnexal tenderness on physical exam with blood in the vaginal vault. The human chorionic gonadotropin level is 6,000 mIU/mL. Her transvaginal ultrasound is shown above. Which of the following gestational ages is the most common time for this condition to present? 10-12 weeks 4-6 weeks 6-8 weeks 8-10 weeks

6-8 weeks ectopic pregancy

36-year-old G1P0 woman presents to the clinic for a prenatal evaluation. Recently, she has had severe nausea and vomiting. Physical exam reveals a larger than expected uterine size for gestational age, with bilateral adnexal masses noted on bimanual exam. Ultrasound imaging of the uterus is shown above. When is the earliest this condition can be diagnosed? 12 weeks gestation 20 weeks gestation 28 weeks gestation 8 weeks gestation

8 weeks gestational trophoblastic disease - molar pregnancy "snowstorm appearance" on US

Which of the following clinical scenarios is the most concerning for the diagnosis of endometrial cancer? A 35-year-old woman who has mid-cycle breakthrough bleeding and has an endometrial stripe measurement of 8 mm A 45-year-old woman whose last menstrual period was 4 months ago and who reports vaginal bleeding, with an endometrial stripe measurement of 4 mm A 69-year-old woman who presents with abnormal vaginal bleeding and an endometrial stripe measurement of 5 mm A 72-year-old woman with history of long-term tamoxifen use who presents with abnormal vaginal bleeding and has an endometrial stripe measurement of 4 mm

A 69-year-old woman who presents with abnormal vaginal bleeding and an endometrial stripe measurement of 5 mm endometrial thickening >5mm and postmenopausal between 50-70 years old

27-year-old woman presents to her primary care physician with concern for postcoital pain and bleeding. She reports no fevers or abdominal pain. She is having normal periods. She is sexually active with men only and has had seven lifetime partners, with one new active sexual partner in the last 2 months. They do not always use a condom. On evaluation, a friable cervix is noted with a few shallow ulcerations on her vaginal tissues. She is tender in these areas but has no cervical motion tenderness. There is no active discharge or concerning odor. She otherwise appears well and reports she has never had these symptoms before. Which of the following is the best choice for treatment? Acyclovir 10 mg/kg/dose IV every 8 hours Acyclovir 400 mg oral three times daily Acyclovir 400 mg oral twice daily Acyclovir topical five times daily

Acyclovir 400 mg oral three times daily cervicitis- HSV

28-year-old G3P2002 woman presents to labor and delivery at 33 weeks and 1 day gestational age complaining of high home blood pressure readings. She reports no headache or visual changes. Her serial blood pressures 4 hours apart are 165/115 mm Hg and 173/102 mm Hg. Her urine dipstick shows 3+ protein. Laboratory tests reveal a creatinine of 0.9 mg/dL and a platelet count of 160,000/µL of blood. Which of the following is the best management? Administration of betamethasone and intravenous labetalol and admission for cesarean delivery Administration of betamethasone and intravenous labetalol and admission for expectant management Administration of betamethasone and intravenous labetalol and admission for vaginal delivery Administration of intravenous labetalol and admission for vaginal delivery

Administration of betamethasone and intravenous labetalol and admission for vaginal delivery *disagree between vaginal delivery or expectant management

28-year-old otherwise healthy pregnant woman presents to the emergency department at 20 weeks gestation with right flank pain. Her other symptoms include dysuria and chills. Vital signs are T of 101.6°F, HR of 114 bpm, BP of 120/80 mm Hg, RR of 20/min, and oxygen saturation of 98% on room air. Physical exam findings include right-sided costovertebral angle tenderness. Urinalysis shows pyuria and bacteriuria. Which of the following is the recommended management? Admission and initiation of intravenous ceftriaxone Admission and initiation of intravenous meropenem Discharge with oral fosfomycin Discharge with oral nitrofurantoin

Admission and initiation of intravenous ceftriaxone acute pyelonephritis pregnant = admission with IV antibiotics (ceftriaxon mc) (meropenem - for immunocompromised and pregnant)

30-year-old nulliparous woman presents with cyclical pelvic pain that has progressively worsened over the last 10 months. She also complains of dysmenorrhea, hematuria, and pain with sexual intercourse. She has been trying to get pregnant but has not been successful. Which of the following physical findings is most suggestive of the suspected diagnosis? Adnexal masses Atrophic vulvar changes Cervical motion tenderness Tender introital mass

Adnexal masses endometriosis

65-year-old woman presents to the clinic with a painless breast lump. Which of the following is the most important risk factor for the most concerning diagnosis? Age Early menarche Late menopause Nulliparity

Age "most significant RF" - women in their 70s

35-year-old G2P1 woman at 16 weeks gestation presents to her obstetrician for a routine visit. Her medical history is significant for gestational diabetes and preeclampsia with her first pregnancy, chronic hypertension, and hypothyroidism. A quadruple test is ordered. Which of the following tests make up the quadruple test? Alpha-fetoprotein, human chorionic gonadotropin, estriol, and inhibin A Parathyroid hormone, thyroid-stimulating hormone, serum thyroxine, and serum triiodothyronine Serum glucose, serum protein, urine glucose, and urine protein Wet prep, Pap smear, human papillomavirus testing, and nucleic acid amplification testing

Alpha-fetoprotein, human chorionic gonadotropin, estriol, and inhibin A done 15-18 wks, screen for genetic abnormalities (esp Down syndrome), neural tube defects, and abdominal wall defects

32-year-old woman who is 37 weeks gestation presents to the maternity ward with a fever of 102.1°F and a heart rate of 115 bpm. She states that she has felt some leaking fluid from her vaginal area for the past 2 days but was unsure of what it was. A pelvic exam reveals fluid leakage from the cervical os. A fetal ultrasound reveals baseline fetal tachycardia. Which of the following antibiotic regimens is the treatment of choice for this patient? Ampicillin and clindamycin Ampicillin and gentamicin Clindamycin and metronidazole Metronidazole and erythromycin

Ampicillin and gentamicin chorioamnionitis RF: PROM (>18 hours) Sx: maternal fever, maternal leukocytosis, purulent fluid leakage, fetal baseline tachycardia

24-year-old G2P1 woman at 32 weeks gestation presents to the ED with menstrual-like cramps, low back pain, vaginal pressure, and light vaginal bleeding. She reports no vaginal pain or drug use but smokes two packs of cigarettes each week. Uterine contractions are occurring 5 times every 20 minutes. Her vital signs are T 98.6°F, BP 136/84 mm Hg, HR 88 bpm, RR 20 breaths per minute, and pulse oximetry 98% on room air. Speculum examination reveals cervical dilation of 4 cm without any pooling of fluid into the posterior vaginal fornix. A fetal fibronectin test is positive. Her urinalysis is unremarkable. Fetal heart rate is 151 bpm, and the nonstress test is reactive. Rectovaginal swab for group B streptococcal testing is obtained. Which of the following clinical interventions is the most appropriate at this time? Ampicillin, betamethasone, and magnesium sulfate Betamethasone and magnesium sulfate Cervical cerclage Eme

Ampicillin, betamethasone, and magnesium sulfate preterm labor bed rest, oral or intravenous hydration, antenatal corticosteroid therapy (e.g., betamethasone) to enhance lung maturity, group B streptococcal infection prophylaxis (e.g., penicillin, ampicillin) until group B streptococcal testing comes back negative or the patient delivers, antibiotic treatment of any documented urinary tract or sexually transmitted infections, magnesium sulfate for neuroprotection (in patients 24-32 weeks gestation), Tocolytic drugs (e.g., indomethacin, nifedipine, magnesium sulfate, terbutaline) may be used to delay delivery for up to 48 hours to allow the antenatal corticosteroid to take full effect.

Which type of pelvis is characterized by convergent side walls of the pelvic midcavity, forward inclination of the sacrum, and a narrow subpubic arch of the pelvic outlet that will most likely lead to labor arrest? Android Anthropoid Gynecoid Platypelloid

Android narrow forepelvis convergent side walls forward inclination of sacrum narrow subpubic arch

26-year-old nulliparous woman presents with her husband to her gynecologist with concerns about family planning. She states they have been unsuccessfully attempting to conceive for 2 years. Which of the following is the most common cause of the patient's condition? Anovulation Decreased sperm count Endometriosis Spontaneous abortion

Anovulation MCC of infertility

22-year-old woman presents to the emergency department with a complaint of acute-onset abdominal pain. She has a low-grade fever, but her vital signs are otherwise normal. On exam, her abdomen is tender, and she has cervical motion tenderness. Purulent vaginal discharge is also noted. She reports being sexually active with multiple casual partners. Her partners do not always use condoms nor does she know much about their medical or sexual histories, but she reports being treated for sexually transmitted infections herself "a few times." A preliminary workup is started for sexually transmitted infections, and a pelvic ultrasound is shown above. Hospital admission is initiated. Which of the following is the best choice for treatment on admission? Antibiotic therapy alone Invasive surgery Minimally invasive drainage Observation

Antibiotic therapy alone (initially) PID (US shows tube-ovarian abscess) Cefotetan and Doxycycline MC used

30-year-old woman presents to the office after a positive home pregnancy test. She has been trying to conceive for 3 months. During that time, she has performed at-home ovulation indicator testing using test strips. She tests on days 7-21 of her cycle. Her last two menstrual cycles were 28 days long. She began tracking ovulation on May 6. During the first month of trying to conceive, she had a positive ovulation test on May 10 and menses began on May 25. During the second month, she had a positive ovulation test on June 12 and menses began on June 22. During the third month of trying to conceive, she had a positive ovulation test on July 7 and menses began on July 17. She had a positive ovulation test on July 30 and a positive pregnancy test on August 12. What is the estimated date of delivery using the Naegele rule? April 21 April 22 April 24 April 30

April 24 subtract 3 months and add 7 days

24-year-old G1P1001 woman is preparing for hospital discharge after a spontaneous vaginal delivery with no complications. Which of the following is the most appropriate recommendation to give her regarding when she can return to sexual activity? As soon as the woman feels ready after a minimum of 2 weeks As soon as the woman feels ready after a minimum of 4 weeks As soon as the woman feels ready after a minimum of 6 weeks As soon as the woman feels ready after a minimum of 8 weeks

As soon as the woman feels ready after a minimum of 2 weeks

Which of the following maternal characteristics is a risk factor for preterm labor? Asthma Body mass index over 30 kg/m2 Prolonged interpregnancy interval (> 5 years) Sedentary lifestyle

Asthma

25-year-old woman presents to her gynecologist with irritability, anxiety, depressed mood, breast pain, bloating, and headaches. She states that her symptoms consistently begin at the same time during her menstrual cycle and last for 5 days before resolving. Which pattern of symptom onset best supports the suspected diagnosis? At the beginning of the follicular phase At the beginning of the luteal phase At the end of the follicular phase At the end of the luteal phase

At the end of the luteal phase PMS

19-year-old woman presents to the urgent care with symptoms of a urinary tract infection. A urine specimen is obtained, and microscopic examination reveals the finding shown in the image above. A pelvic examination is subsequently performed and reveals an inflamed cervix with punctuate hemorrhages and frothy green discharge. Which of the following should the patient be informed about regarding her treatment? Avoid alcohol Night terrors Red-orange discoloration of urine Tendon rupture

Avoid alcohol trichomoniasis - metronidazole (Flagyl) 2g PO

65-year-old woman presents to the clinic with right-sided abdominal fullness and early satiety that has developed insidiously over the past 2 months. Physical examination reveals right-sided adnexal fullness. Laboratory testing shows a cancer antigen 125 of 300 units/mL. Transvaginal ultrasound shows a right-sided adnexal mass with solid components and irregular septations. Which of the following is a risk factor for the most likely diagnosis? BRCA1 genetic mutation History of breastfeeding Oral contraceptive use Tubal ligation

BRCA1 genetic mutation Ovarian Cancer

25-year-old G1P0 woman presents to her obstetrician for her first prenatal visit. She would like to know what kind of testing will be done at each visit throughout her pregnancy. Which of the following should be obtained at every visit? Blood pressure Hemoglobin level Serum protein level Urine culture

Blood pressure

53-year-old woman presents to the clinic with hot flashes, difficulty sleeping, and vaginal dryness. She reports the symptoms have progressively gotten worse over the past 3 years. She reports no surgical history. Which of the following cancers would this patient be at significant increased risk of if she is treated with combined menopausal hormone therapy? Breast Colorectal Endometrial Ovarian

Breast

68-year-old woman presents to her primary care office reporting abdominal bloating that has worsened over the last 4 months. Her abdomen appears distended and has a significant fluid wave on physical exam. On pelvic exam, her cervix appears normal, however, it is slightly shifted towards the left. During the bimanual exam, there is fullness and what feels like a mass to the right adnexa. Which of the following tumor markers would you expect to be elevated with this patient's diagnosis? Alpha-fetoprotein CA 125 CA 15-3 CA 19-9

CA 125 (ovarian cancer) AFP - liver cancer CA 15-3 - breast cancer CA 19-9 - gallbladder or pancreatic cancer

65-year-old woman presents to her gynecologist after a visit to the ED. The patient reports that she was having abdominal pain last night and went to the ED. Her abdominal pelvic CT scan showed a 10 cm solid, right ovarian mass with multiple small peritoneal masses. Which of the following is most likely to be increased? Cancer antigen 125 Cancer antigen 15-3 Carbohydrate antigen 19-9 Carcinoembryonic antigen

Cancer antigen 125 Ovarian Cancer(>35 U/mL)

23-year-old woman presents to the emergency department with bilateral lower abdominal pain, severe vomiting, and vaginal discharge for 4 days. She has had no recent gynecologic instrumentation. Vital signs are T of 102.6°F, HR of 125 bpm, BP of 128/74 mm Hg, RR of 20/min, and oxygen saturation of 98% on room air. Physical exam reveals cervical motion tenderness. Computed tomography of the abdomen and pelvis is ordered and is unremarkable. The patient is admitted to the hospital for observation. Which of the following is the recommended antibiotic regimen for the most likely diagnosis? Cefoxitin and doxycycline Cefoxitin and metronidazole Ceftriaxone and azithromycin Ceftriaxone and doxycycline

Cefoxitin and doxycycline PID

24-year-old pregnant woman presents to the clinic with vaginal itching and discharge. She recently had unprotected intercourse with a new sexual partner. The Gram stain from microscopy is shown above. Which of the following is the recommended treatment? Azithromycin and gemifloxacin Ceftriaxone Ceftriaxone and azithromycin Ceftriaxone and doxycycline

Ceftriaxone and azithromycin gonorrhea in pregnancy (use doxy in not pregnant)

21-year-old woman presents to her gynecologist extremely upset and scared. She states she was at a fraternity party last night and blacked out. When she woke up this morning, she was completely unclothed. She reports vaginal pain and bleeding. The patient is up-to-date on her hepatitis B and human papillomavirus (HPV) vaccinations. In addition to offering prophylactic treatment for HIV, which of the following antibiotics would be most appropriate for empiric coverage of sexually transmitted infections? Azithromycin, levofloxacin, metronidazole Ceftriaxone, ciprofloxacin, metronidazole Ceftriaxone, doxycycline, metronidazole Ceftriaxone, levofloxacin, metronidazole

Ceftriaxone, doxycycline, metronidazole gonorrhea, chlamydia, trich coverage

26-year-old G4P0121 woman presents to the emergency department at 15 weeks gestation with painless vaginal spotting. She reports she has had two miscarriages during the second trimester that presented similarly. Transvaginal ultrasound confirms cervical dilation and reveals a cervical length of 20 mm. Which of the following is the recommended treatment? Abstinence from intercourse Bedrest Cervical cerclage Progesterone supplementation

Cervical cerclage cervical insufficiency

28-year-old G2P1 woman presents to the clinic at 32 weeks gestation for a routine prenatal visit. She has an ultrasound performed to monitor a previously diagnosed placental condition, and it shows persistent homogeneous placental tissue extending over the internal cervical os. The patient reports no vaginal bleeding during her pregnancy. Which of the following is the most appropriate recommendation regarding delivery, assuming the underlying condition persists and the patient remains asymptomatic? Cesarean delivery at 37 weeks Cesarean delivery at 40 weeks Induction of vaginal delivery at 37 weeks Induction of vaginal delivery at 40 weeks

Cesarean delivery at 37 weeks Placenta previa

24-year-old G2P2 woman in the labor and delivery unit develops fever, lower abdominal pain, and foul-smelling, purulent lochia 3 days after a cesarean section for prolonged rupture of membranes. Her vital signs are T of 38.4°C (101.1°F), BP of 130/70 mm Hg, HR of 105 bpm, RR of 18 breaths per minute, and oxygen saturation of 98% on room air. Her pregnancy was complicated by bacterial vaginosis and heavy vaginal colonization with Escherichia coli. Which of the following is the most significant risk factor for the patient's current condition? Bacterial vaginosis Cesarean section Heavy vaginal colonization with Escherichia coli Prolonged prelabor rupture of membranes

Cesarean section most important RF for postpartum endometritis

26-year-old G4P3 woman presents at 40 weeks gestation for induction of labor. She has a history of type 2 diabetes mellitus, and her body mass index is 37 kg/m2. During the second stage of labor, the fetal head delivers then retracts into the perineum. Gentle downward traction applied to the fetal head, while the patient's legs are elevated and her thighs brought to her abdomen, results in tearing of the perineum but is unsuccessful in delivering the infant. Maneuvers to deliver the posterior shoulder and placing the patient on her hands and knees are also unsuccessful. The obstetrician places the fetal head back into the pelvis. What is the best next step in the management of this patient? Applying more downward traction with simultaneous fundal pressure Cesarean section Clamping and cutting the umbilical cord Episiotomy

Cesarean section shoulder dystocia

60-year-old woman presents with a complaint of vaginal fullness that is exacerbated by prolonged standing and relieved by lying down. On exam, you note a soft, reducible mass protruding into the introitus. Which of the following is the most common risk factor for the suspected diagnosis? Aging Childbirth and pregnancy Frequent constipation Heavy lifting

Childbirth and pregnancy

Which of the following is the most common cause of cervicitis? Bacterial vaginosis Chlamydia trachomatis Neisseria gonorrhoeae Trichomonas vaginalis

Chlamydia trachomatis MC STI

25-year-old woman presents to the clinic for follow-up after testing was performed to evaluate her persistent vaginal bleeding and hemoptysis. She had a vaginal delivery of a term intrauterine pregnancy 4 months ago. Her hCG level is 100,000 mIU/mL. Transvaginal ultrasound shows an enlarged uterus without a fetal heartbeat. Chest X-ray shows multiple pulmonary nodules. Subsequent biopsy of the pulmonary nodules shows sheets of trophoblastic tissue consisting of syncytiotrophoblasts and cytotrophoblasts without villi. Which of the following is the most likely diagnosis? Choriocarcinoma Invasive mole Placental-site trophoblastic tumor Uncomplicated complete molar pregnancy

Choriocarcinoma most follow previous pregnancy, spreads hematogenously, common distant metastasis sheets of trophoblastic tissue consisting of synchytiotrophoblasts and cytotrophoblasts without villi

23-year-old woman presents to her gynecologist due to fever, nausea, vomiting, and lower abdominal pain. She is currently sexually active and uses condoms occasionally. Vital signs are T of 38.4°C (101.1°F), BP of 138/78 mm Hg, HR of 90 bpm, RR of 18 breaths per minute, and oxygen saturation of 99% on room air. Physical examination is significant for bilateral lower abdominal tenderness to palpation. Pelvic examination is significant for purulent endocervical discharge, cervical motion tenderness, and uterine tenderness without adnexal masses. The patient is sent to the ED and admitted for treatment. Which of the following is a complication of the most likely diagnosis? Cervical cancer Chronic pelvic pain Endometriosis Ovarian torsion

Chronic pelvic pain PID

A 32-year-old woman who is 24-hours post-op after delivering her first child via C-section suddenly develops abdominal pain, foul-smelling vaginal discharge, and uterine tenderness. Her vitals are O2 saturation at 99% on room air, blood pressure at 132/86 mm Hg, heart rate at 115 beats per minute, and temperature 102°F. What is the appropriate treatment for this patient's suspected diagnosis? Ampicillin and gentamicin Clindamycin and gentamicin Doxycycline and ceftriaxone First-generation cephalosporin alone

Clindamycin and gentamicin endometritis after C-section C-sections prophylactically given a dose of a first-generation cephalosporin during occurs after a vaginal delivery- ampicillin and gentamicin

28-year-old woman presents to the clinic due to infertility. She has infrequent menstrual cycles and has been unable to become pregnant despite 12 months of frequent unprotected intercourse. Physical examination is unremarkable other than hirsutism and inflammatory acne. Her husband has been evaluated and had a normal semen analysis. Which of the following is the recommended treatment for infertility in this patient? Clomiphene citrate In vitro fertilization Intrauterine insemination Leuprolide

Clomiphene citrate an ovulation due to PCOS

36-year-old G2P1 woman presents for a routine obstetrics visit. She feels well and has no complaints. She reports occasional instances of heartburn and has to wear compression stockings daily while at work to prevent lower extremity swelling and discomfort. She feels occasional pelvic pressure and has been urinating more frequently. She reports no burning with urination, flank pain, or hematuria. Yesterday, she felt a gush of fluid vaginally and had to rush to the bathroom because she was worried about incontinence. Since then, she has been leaking a "very small amount of clear-yellow urine." What result of nitrazine testing will confirm her diagnosis? Color change to blue Ferning on microscopy pH of 4.0 Strong fishy odor

Color change to blue nitrazine - test pH of amniotic fluid: 7-7.3 (will turn paper blue)

67-year-old woman presents with a persistent vulvovaginal dermatitis. Her symptoms have persisted for 1 month. She reports frequent vulvar itching and burning. She has a longstanding history of vulvar erythema, which is usually not bothersome when she uses a barrier cream. She has treated the area with over-the-counter monistat and athlete foot medication without relief in symptoms for the last 6 weeks. Last week, she took fluconazole 150 mg in a single dose and began treating the symptomatic area with clotrimazole-betamethasone 1% topical cream twice a day. On vulvar exam, she has erythema and scaling bilaterally, which appears to be unchanged since her last visit. The above image details her physical exam findings. Which of the following is the best next intervention? Colposcopic biopsy of the vulva Human papillomavirus testing Potassium hydroxide preparation Vulvar culture

Colposcopic biopsy of the vulva vulvar cancer -- Paget disease (adenocarcinoma of apocrine and eccrine glands) *similar to candidiasis but will not resolve with tx* Sx: pruritus, soreness, irritation and burning, redness, thickened scaly skin

35-year-old woman presents for a follow-up with her gynecologist. She had a Pap smear completed 2 weeks ago, and the results were positive for human papillomavirus with low-grade squamous intraepithelial lesions. Which of the following is the most appropriate next step in management? Colposcopy Endocervical curettage Follow-up in 1 year for a repeat Pap smear and human papillomavirus testing Loop electrosurgical excision procedure

Colposcopy performed on patients with a positive HPV test with ASC-US, HSIL, LSIL, or atypical glandular cells If abnormal areas are seen, then punch biopsy or endocervical curettage should be obtained for further tissue evaluation.

You are reviewing results from a 31-year-old woman who underwent recent cervical cancer screening. Her cervical cytology shows atypical squamous cells of undetermined significance, and her human papillomavirus test is positive. Her pelvic exam does not reveal any gross cervical lesions. Which of the following is the recommended next step for her? Colposcopy Colposcopy with endocervical sampling Loop endocervical excisional procedure Repeat cytology in 1 year

Colposcopy with endocervical sampling sampling recommended who do not have visible lesions on physical examination

28-year-old woman with a past medical history of diabetes mellitus presents to the clinic with irregular menstrual cycles. She says her cycles occur every 2 months. Physical examination reveals male-pattern facial hair and inflammatory acne. Which of the following is the recommended treatment for her hyperandrogenic symptoms? Clomiphene Combined oral contraceptive pill Levonorgestrel-releasing intrauterine device Spironolactone

Combined oral contraceptive pill PCOS 1st line: weight loss pharm: combine OCP spirolactone --> adjunct if no improvement after 6 months Clomiphen --> tx infertility to induce ovulation

34-year-old woman is admitted in the hospital for routine care following a vaginal delivery. She plans on breastfeeding. You are counseling her on contraceptive options during the postpartum period. Which of the following contraceptive options is contraindicated during the postpartum period? Combined oral contraceptives Depot medroxyprogesterone acetate injection Etonogestrel implant Levonorgestrel-releasing intrauterine device

Combined oral contraceptives *increased risk of venous thromboembolism*

A 38-year-old G5P4 woman is in active labor. She is 6 cm dilated and 100% effaced. On vaginal exam, a smooth, soft mass is palpated, and two feet can be identified as the presenting parts. An abdominal ultrasound is ordered and reveals the knees tucked up against the fetal chest and the feet and buttocks at the birth canal. Which fetal presentation is being described? Complete breech Frank breech Incomplete breech Nonfrank breech

Complete breech

22-year-old woman is in labor, and you are watching the fetal heart rate monitor waveform. You notice the fetal heart rate gradually decreases at the start of the uterine contraction, reaches a nadir at the peak of the contraction, and returns to baseline as the contraction ends. Which of the following is the most likely cause of this change in the fetal heart rate? Compression of the fetal head Compression of the umbilical cord Fetal movement Uteroplacental insufficiency

Compression of the fetal head early decel

24-year-old woman presents with a headache, fever, and an erythematous rash on the flanks that she noticed yesterday. She reports that she had a painless vaginal ulcer that resolved 5 weeks ago. On exam, you note a maculopapular rash on the trunk as well as on the palms and soles. Which of the following additional physical findings is most likely to be present? Bubo Chancre Condyloma latum Gumma

Condyloma latum (wart-like lesions) secondary syphilis Bubo - tender inguinal or femoral lymphadenopathy (lymphogranuloma venereum - chlamydia) Chancre - painless, crater-like lesion (primary syphilis) Gumma - soft, noncancerous growth on internal organs (tertiary syphilis)

A 22-year-old woman presents to the office due to gential bumps that are pruritic and not painful. Physical exam reveals multiple papillomatous skin-colored lesions, as seen on the image above, on the labia and perianal area. Which of the following is the most likely diagnosis? Condylomata acuminata Condylomata lata Molluscum contagiosum Papulosquamous lichen planus

Condylomata acuminata HPV 6 and 11, cauliflower genital warts

28-year-old otherwise healthy woman presents to the gynecology clinic with 24 hours of right-sided breast pain, redness, and swelling. She is 5 weeks postpartum. Vital signs are T of 100.6°F, HR of 98 bpm, BP of 120/80 mm Hg, RR of 20/min, and oxygen saturation of 98% on room air. Breast examination reveals a local area of erythema and firm edema on the right breast. Which of the following is the recommended treatment? Cessation of breastfeeding and clindamycin Cessation of breastfeeding and dicloxacillin Continuation of breastfeeding and clindamycin Continuation of breastfeeding and dicloxacillin

Continuation of breastfeeding and dicloxacillin lactational mastitis

25-year-old G1P0 woman at 28 weeks gestation presents to her obstetrician for a routine visit. Her blood pressure was previously normal, until her 24-week visit when it was measured at 142/92 mm Hg. Today, her blood pressure is 144/92 mm Hg. She reports no vision changes, headache, or abdominal pain. Urinalysis is significant for 2+ protein. Complete blood count and kidney function testing is unremarkable. Which of the following is the most appropriate treatment at this time? Aliskiren Continued monitoring Labetalol Losartan

Continued monitoring Antihypertensive therapy: systolic blood pressure of ≥ 160 mm Hg or diastolic blood pressure of ≥ 110 mm Hg

A 27-year-old woman presents to her gynecologist with a left-sided breast mass. Physical examination reveals the presence of a 2 cm, firm, immovable, single lesion on the left breast at the 11 o'clock position. An ultrasound reveals a well-visualized, hypoechogenic 2 cm tall x 1.5 cm wide irregular lesion. Which of the following is the most appropriate diagnostic study at this time? Core-needle biopsy Fine-needle aspiration Mammography Surgical biopsy

Core-needle biopsy preferred initial biopsy for suspicious masses

8-year-old woman presents to the clinic with recurrent and pelvic pain each month with the onset of menses. Which of the following best describes the classic pain of the most likely diagnosis? Crampy and midline Crampy and unilateral Dull and midline Dull and unilateral

Crampy and midline dysmenorrhea

A 27-year-old woman who is positive for HIV presents to the clinic with bumps near her anus. On exam, there are skin-colored cauliflower lesions on the perianal skin. Which of the following is the best clinician-administered treatment? Cryotherapy Imiquimod Podophyllotoxin Rubber band ligation

Cryotherapy condyloma acuminata (genital warts, HPV 6&11) *patient-applied* options include imiquimod, podophyllotoxin, and sinecatechins.

A 35-year-old woman presents to a women's health clinic requesting oral contraceptive therapy. The physician assistant decides to prescribe a combined estrogen-progestin hormonal contraceptive. Which of the following is a relative contraindication to combined estrogen-progestin oral contraception for this patient? Body weight > 198 lbs (90 kg) Current tobacco use of 10 cigarettes/day History of stroke Migraine with aura

Current tobacco use of 10 cigarettes/day History of stroke (C) and migraine with aura (D) are *absolute* contraindications

A 68-year-old multiparous woman presents to the clinic with complaints of vaginal fullness and urinary incontinence. She reports no recent pelvic trauma or surgeries. On exam, you note a vaginal bulge through the introitus and soft anterior fullness of the vaginal wall. Urinalysis reveals a few normal epithelial cells. A funnel-shaped bladder is seen on transabdominal ultrasound. Which of the following is the most likely diagnosis? Bladder fistula Cystocele Rectocele Urinary tract infection

Cystocele

21-year-old woman presents to the office with a bump on her labia that appeared 3 days ago. It is not painful to touch nor pruritic. She had unprotected intercourse with a man about 3 weeks ago and has not had any sexual contact with anyone else since. On physical exam, there is a 2 cm nontender, ulcerated, round, skin-colored lesion with an elevated rim that is located on the inside of the right labium majus. There are also palpable lymph nodes to the inguinal area bilaterally. Which of the following tests will provide a definitive diagnosis of her condition? Darkfield microscopy Fluorescent treponemal antibody absorption test Microhemagglutination test for antibodies to T. pallidum Venereal Disease Research Laboratory

Darkfield microscopy syphilis caused by T. pallidum

50-year-old woman presents with a complaint of vaginal dryness. She reports having some vulvovaginal irritation, and she has noticed decreased vaginal lubrication during intercourse. Her symptoms have been getting worse over the past 2 years since she went through menopause. The patient is in a monogamous relationship with no concern for sexually transmitted infections. Her pelvic exam reveals atrophic skin changes along the vaginal canal with decreased secretions. No other abnormalities are noted. Which of the following is the most likely cause of this patient's symptoms? Decreased estrogen Decreased progesterone Lichen planus Vaginal yeast infection

Decreased estrogen atrophic vaginitis in menopause

A 25-year-old woman presents to her gynecologist with complaints of painful cyst-like masses in both breasts. She states her breast masses seem to fluctuate in size rapidly and they are the most painful right before her menstrual cycle begins. Ultrasound of the breasts reveals the presence of multiple small cysts bilaterally with no malignant or concerning features. Which of the following lifestyle modifications should be recommended for this patient? Decreasing dietary fat intake Engaging in weighted interval training Increasing caffeine intake Switching from a tight to a loose brassiere

Decreasing dietary fat intake fibrocystic disease

23-year-old woman presents to the emergency department with right-sided abdominal pain, nausea, and vomiting that began suddenly about 30 minutes ago after she went to a group exercise class. She was recently diagnosed with polycystic ovarian disease. Upon physical examination, she does not have any tenderness to palpation to her abdomen or pelvis and there are no palpable masses or evidence of distension. A pregnancy test is negative. Which of the following is the best way to definitively diagnose her condition? Computed tomography of the abdomen and pelvis with contrast Direct visualization at the time of surgical evaluation Magnetic resonance imaging of the abdomen and pelvis with contrast Pelvic ultrasound with Doppler

Direct visualization at the time of surgical evaluation Ovarian Torsion *dumb question

24-year-old woman presents to the emergency department with sudden onset of right-sided pelvic pain associated with nausea. Vital signs are T of 97.8°F, HR of 85 bpm, BP of 132/84 mm Hg, and RR of 20/min and have been stable during the patient's 4 hours in the emergency department. Physical exam reveals no vaginal bleeding and right-sided adnexal tenderness during bimanual exam. Pregnancy test is negative. Hemoglobin is 12.5 g/dL. Pelvic ultrasound reveals a right-sided 4 cm ovarian cyst with mild to moderate surrounding blood in the pelvis. The patient's pain is improved following intravenous ketorolac. Which of the following is the best management? Admission for close observation Discharge with pain control Gynecologic consultation and emergent manual detorsion Gynecologic consultation for emergent laparoscopy to stop the hemorrhage

Discharge with pain control Reasons for admission: vital instability, low hemoglobin, uncontrolled pain, large hemorrhage

28-year-old G2P1 woman at 36 weeks gestation presents to the ED with uterine contractions, abdominal pain, and vaginal bleeding for the past 2 hours. Her blood pressure is 88/60 mm Hg, and her heart rate is 115 bpm. Physical examination is notable for a rigid, tender uterus. Ultrasound is significant for a retroplacental hematoma. Fetal heart rate tracing is nonreassuring. The patient is prepped for an emergent cesarean section. Which of the following is the most common potential complication of the patient's condition? Cardiogenic shock Disseminated intravascular coagulation Septic shock Thrombocytopenic thrombotic purpura

Disseminated intravascular coagulation placental abruption - painful vaginal bleeding

25-year-old woman who is RhD negative presents to the clinic to establish prenatal care. She is 17 weeks pregnant with normal vital signs and positive fetal heart tones. Prior obstetric history is positive for three preterm births, each progressively earlier in gestation, with no living children. The father of her child is RhD positive (homozygous). A serum maternal anti-D titer is 1:32. Which of the following represents the best next step in the prenatal care of the patient and her fetus? Administration of anti-D immune globulin to the mother Doppler velocimetry of the fetal middle cerebral artery Fetal hemoglobin measurement via cordocentesis Serial intrauterine blood transfusions

Doppler velocimetry of the fetal middle cerebral artery --> *used when maternal anti-D titer reached critical titer level (1:32) admin of anti-D globulin is preventative strategy during first pregnancy of incompatibility

27-year-old woman presents to the emergency department with complaints of dull low-back and lower-abdominal pain for the last 2 weeks. She reports no recent trauma, but she did travel to the Caribbean a month ago. She has increased pain with defecation but no constipation. She has had normal menstrual cycles but does seem to have more vaginal discharge than usual. Her sexual history is notable for multiple sexual partners with whom protection is not always used. She reports practicing receptive anal, oral, and vaginal intercourse. She has a history of prior gonorrhea and chlamydia diagnoses and treatments, but she has not seen a clinician for several months. Her exam is notable for tender, enlarged inguinal lymph nodes and a painful anorectal exam with palpation of several indurated tender areas in her rectal canal. A full sexually transmitted infection workup is done, including an anorectal swab sent for nucleic aci

Doxycycline 100 mg oral bid for 21 days chlamydia caused genital ulcer disease (lymphogranuloma venereum)

26-year-old G1P0 woman presents for a prenatal appointment. She is 39 weeks pregnant. On ultrasound, her fetus is in the head-down position and measures 9 lb 6 oz. The fetal heart rate is 155 bpm. Vaginal exam reveals a narrow pelvic outlet. Which of the following should be addressed as a likely result of episiotomy as compared to a spontaneous vaginal laceration when discussing the possibility of a vaginal laceration with the patient? Decreased repeat lacerations with subsequent births Dyspareunia Lower rate of dehiscence Shorter perineal lacerations

Dyspareunia

32-year-old woman presents to the clinic to discuss recurrent miscarriages. She reports three miscarriages that occurred at 18 weeks, 19 weeks, and 21 weeks. She reports that, when she miscarries, she has painless vaginal bleeding without significant contractions. Which of the following is a risk factor for the suspected diagnosis? Cigarette smoking Ehlers-Danlos syndrome Obesity Prior placenta previa

Ehlers-Danlos syndrome cervical insufficiency RF leading to miscarriages

56-year-old G1P1001 woman presents to the clinic with the chief complaint of vaginal bleeding. She states she has not had any bleeding since her last menstrual period 5 years ago. A transvaginal ultrasound shows an 8 mm endometrial stripe. Which of the following is the next most appropriate step in management? Dilation and curettage Endometrial biopsy Hysterectomy Hysteroscopy

Endometrial biopsy endometrial cancer

50-year-old woman presents to her gynecologist with a complaint of intermittent vaginal bleeding for the last 3 months. You note in her chart that she is postmenopausal. She states that it was intermittent spotting initially, but now she has some bleeding daily. She is not currently sexually active and does not complain of pain. Pelvic exam shows scant blood in the cervical os with no abnormalities visualized. The uterus is irregular in size and fixed. Ultrasound shows an endometrial thickness of 6 mm. Which of the following is the most appropriate diagnostic test? CT scan Endometrial biopsy Human chorionic gonadotropin level Magnetic resonance imaging

Endometrial biopsy --> endometrial thickness >4mm endometrial cancer

24-year-old woman presents to the clinic reporting infertility. Physical exam reveals hirsutism, obesity, and a fasting glucose of 210 mg/dL. Pelvic ultrasound reveals multiple simple cysts in each ovary. Which of the following is this patient at increased risk of developing? Breast cancer Endometrial cancer Ovarian cancer Pancreatic cancer

Endometrial cancer PCOS *due to anovulatory cycles cause unopposed exposure to estrogen

28-year-old nulliparous woman presents to the clinic reporting left lower quadrant pain and severe dysmenorrhea. A pelvic ultrasound reveals a left adnexal complex mass that has smooth walls with homogeneous internal echoes that have a ground-glass appearance. Which of the following ovarian masses is the most likely diagnosis? Corpus luteal cyst Dermoid cyst Endometrioma Follicular cyst

Endometrioma smooth-walled with homogenous internal echos with ground-glass appearance chocolate colored on biopsy

17-year-old girl presents to the urgent care with concerns about a new vaginal odor and discharge for the past 2 days. She is currently sexually active and does not use condoms. She reports no dysuria, dyspareunia, vaginal pruritus, or vaginal discomfort but regularly douches and smokes 15 cigarettes per day. Speculum examination reveals a homogeneous, thin, grayish-white discharge on the vaginal walls. No punctate hemorrhages are seen on the cervix. Vaginal pH is 5.0. Which of the following is most likely seen on wet mount, given the most likely diagnosis? Budding yeast and hyphae Epithelial cells with stippled borders Motile trichomonads Numerous white blood cells

Epithelial cells with stippled borders Clue cells of BV

Which of the following is routinely given to neonates at birth to prevent ophthalmia neonatorum in the United States? Erythromycin ophthalmic ointment Povidone-iodine solution Silver nitrate solution Tetracycline ophthalmic ointment

Erythromycin ophthalmic ointment aka baby pink eye other choices are used but not commercially available in US

54-year-old woman presents due to vaginal discomfort. Symptoms began a few years ago and have become progressively worse. She reports daily discomfort and is no longer interested in sexual intercourse because of the pain. Her medical history includes seasonal allergies, for which she takes daily loratadine. Her past surgical history includes a tonsillectomy 10 years ago, partial hysterectomy 3 years ago, and a cholecystectomy 2 years ago. She reports no hot flashes, insomnia, mood swings, or difficulty with memory. A vaginal exam reveals sparse pubic hair, a narrowed vaginal introitus, atrophic vaginal rugae, and pale vaginal tissue. Which of the following is the most appropriate treatment option for this patient? Combination estradiol 1 mg and progesterone 100 mg oral tablet Estradiol 0.01% vaginal cream Estradiol 2 mg oral tablet Paroxetine 20 mg tablet

Estradiol 0.01% vaginal cream for specifically vaginal sx of menopause

52-year-old woman presents with complaints of severe hot flashes and insomnia. She reports that they have been getting worse over the past 6 months and that it has been 14 months since her last period. The patient does not have any medical conditions or surgical history. What would be the recommended treatment for this patient? Estrogen Estrogen and progestin Testosterone Venlafaxine

Estrogen and progestin *progestin to oppose estrogen to protect against endometrial cancer

25-year-old woman presents to the clinic for a routine wellness exam. She is considering contraceptive options. Which of the following contraceptive options has a black box warning due to the risk of blood clots? Copper intrauterine device Ethinyl estradiol and norelgestromin patch Etonogestrel implant Medroxyprogesterone acetate injection

Ethinyl estradiol and norelgestromin patch *combined hormonal* inc risk for embolism even higher in patch vs pill due to higher concentration in patch

24-year-old woman presents to the gynecology clinic for evaluation of breast tenderness and bloating that occurs the week before menses. She also says she is irritable, and the symptoms together make it difficult for her to work. These symptoms occur consistently month to month. Which of the following changes may help with this condition? Exercising consistently Increasing salt Reducing red meats Reducing starches

Exercising consistently PMS

30-year-old pregnant woman with a history of uncontrolled asthma presents to the obstetrics clinic at 30 weeks gestation for a routine prenatal visit. Her blood pressure measures 152/108 mm Hg and 154/106 mm Hg several hours later on recheck. She has no history of hypertension, and she reports no chest pain, shortness of breath, headache, vision changes, or abdominal pain. Laboratory studies include a urine dipstick negative for protein, 180,000 platelets/µL, creatinine of 0.7 mg/dL, aspartate aminotransferase of 25 U/L, and alanine aminotransferase of 28 U/L. Which of the following antihypertensives is the most appropriate choice to initiate in this patient? Extended-release nifedipine Labetalol Lisinopril Losartan

Extended-release nifedipine gestational hypertension *labetalol also used in pregnancy BUT ci with uncontrolled asthma

24-year-old G2P1 woman is in the second stage of labor. She is 10 cm dilated and 100% effaced. Fetal station is +3. She reports feeling the urge to push, and she is most comfortable in the supine position. With her next contraction, she pushes for three 10-second intervals. The fetal head is delivered and a nuchal cord is freed. Which of the cardinal movements of labor will be the next to occur? Expulsion Extension External rotation Flexion

External rotation moving from face-down position to facing one of the thighs - help the shoulder through pelvic arch

60-year-old G6P6 woman presents to the gynecology clinic complaining of pelvic pressure and vaginal bulging. She also reports a sensation of incomplete emptying following bowel movements. Physical examination reveals prolapse at the posterior vaginal wall. Which of the following additional findings is most consistent with the most likely diagnosis? Complete urinary obstruction Fecal incontinence Stress urinary incontinence Urinary frequency

Fecal incontinence Rectocele

37-year-old G4P1012 woman at 12 weeks gestation presents to the clinic with abdominal pain and vaginal bleeding. The patient states that she had a sudden episode of vaginal bleeding yesterday and has since had abdominal pain with vaginal spotting. Vital signs are HR 74 bpm, BP 120/72 mm Hg, and RR 16 breaths per minute. Pelvic exam reveals opened cervical os with evidence of fetal tissue. What is the most common cause of the suspected diagnosis? Autoimmune disorders Fetal chromosomal abnormalities Maternal malnutrition Uncontrolled diabetes mellitus

Fetal chromosomal abnormalities MCC of spontaneous abortion

28-year-old woman who is G1P0 and 30 weeks pregnant presents to the labor and delivery department reporting pelvic pressure and some mild contractions that vary from 5 to 15 minutes apart. Fetal heart sounds are normal, and there is no pooling of fluid in the vaginal vault. Which of the following tests should be ordered to predict the risk of preterm delivery? Amniocentesis Fetal fibronectin test Group B Streptococcus culture Nitrazine test

Fetal fibronectin test if this test is negative (less than 50 ng/mL), it has a high negative predictive value, indicating the patient will not give birth for another 7-14 days

17-year-old woman presents to the office with concerns about a new breast mass. She reports the mass has been present for the last 4 months and seems to get larger around the time of her period. The mass is tender when it is enlarged. It usually regresses back to its original size after each menstrual cycle and is no longer tender. Physical exam reveals a well-circumscribed, smooth, rubbery, mobile 1.8 cm mass in the right upper quadrant of the right beast. It is nontender, and there are no overlying skin changes. There is no axillary lymphadenopathy. What is the most likely diagnosis? Breast cancer Breast cyst Fibroadenoma Lipoma

Fibroadenoma MC benign in age 15-35

35-year-old woman presents to her gynecologist with complaints of breast pain. The patient reports increased breast pain and swelling for 3 days prior to her menstrual cycle for the past several years. She became concerned when she noticed a mass in her right breast. Physician exam is positive for a small, fixed lesion in the upper outer quadrant of the right breast. Ultrasound shows a 1 cm lesion, and biopsy is recommended. Core needle biopsy reveals tissue nonproliferation with fibrotic changes. What is the most likely diagnosis for this patient? Breast cancer Fibrocystic disease Intraductal papilloma Radial scar

Fibrocystic disease

25-year-old G2P1 pregnant woman is in labor. Her cervix is dilated to 7 cm and has been dilating quickly. Which of the following stages of labor is she in? First Fourth Second Third

First active phase of first stage (rapid cervical changes, begins approx. 6cm)

During vaginal delivery, the clinician notices slight retraction of the fetal head after it has been delivered. Gentle, downward traction on the fetal head fails to deliver the anterior shoulder. Which of the following techniques should be implemented initially to deliver the fetus? Deliver the posterior shoulder Flex the maternal thighs toward the abdomen Return the fetal head to the uterine cavity and proceed with cesarean delivery Rotate the fetus using pressure over the posterior shoulder clavicle

Flex the maternal thighs toward the abdomen McRoberts maneuver - first to be attempted in shoulder dystocia *2nd line: Menicolgou - delivery of posterior shoulder

31-year-old woman presents to her clinician. She and her partner are hoping to start a family soon, and she would like advice about having a successful and healthy pregnancy. Her family history is notable for asthma, and she has a personal history of hypothyroidism. Her partner has a child with spina bifida from a previous marriage. They are both otherwise healthy and make deliberate efforts to eat healthy and exercise. Which of the following recommendations should be made for supplementation during pregnancy for this patient, in addition to other recommendations for preconception health? Folic acid 0.4 mg daily Folic acid 4 mg daily Vitamin A 10,000 IU daily Vitamin A 5,000 IU daily

Folic acid 4 mg daily reduce risk of neural tube defects *Vit A is teratrogenic*

A 44-year-old woman presents to her gynecologist for her annual visit and raises concerns about recent irregularities to her menstrual cycle. She reports no sexual activity in the last year. She has not been experiencing hot flashes. She does feel moodier than usual but also reports not sleeping well recently and does not think this is related since she just changed jobs. She is otherwise well and reports no abdominal pain or abnormal bleeding. Which of the following is the best test to evaluate whether her symptoms are due to the early onset of menopause? Antimüllerian hormone Clomiphene citrate challenge test Follicle-stimulating hormone Luteinizing hormone

Follicle-stimulating hormone Menopause - low estrogen, high FSH

25-year-old pregnant woman presents to the obstetric clinic at 8 weeks gestation with dysuria for 2 days. She has had urinary frequency and urgency for the past 3 weeks. She reports no flank pain. Physical examination reveals no costovertebral angle tenderness. Urinalysis shows pyuria. Which of the following is the recommended treatment? Ciprofloxacin Fosfomycin Nitrofurantoin Supportive treatment with acetaminophen

Fosfomycin antibiotics (fosfomycin, amoxicillin-clavulanate, cefpodoxime) tx for cystitis

42-year-old G3P2 woman presents in early labor. She is 39 weeks pregnant. Her first two pregnancies presented cephalically and were delivered vaginally at term without complication. One child was 8 pounds, 12 ounces and the other was 9 pounds, 7 ounces. Apgar scores at 1 and 5 minutes were 8 and 9 for both children. Her current pregnancy has progressed normally. Her glucose screening test performed at 26 weeks gestation was elevated at 151 mg/dL. Her 3-hour glucose test returned within normal limits. She is obese and has gained the appropriate amount of weight during this pregnancy. On physical exam, she is in slight discomfort. Contractions are occurring regularly every 5 minutes. She is 3 cm dilated and 100% effaced. The fetus is in frank breech presentation. Which aspect of the patient's history and physical exam increases the risk of a prolapsed umbilical cord? Frank breech presentation History of abnormal glucos

Frank breech presentation

34-year-old woman has four previous pregnancies and is currently 12 weeks pregnant. In the past, she has delivered at 37 weeks, 35 weeks, and 39 weeks, and she had a miscarriage at 7 weeks. She has four living children. Which of the following accurately describes her obstetrics history? G4P1214 G5P2113 G5P2114 G6P2214

G5P2114 G: total number of pregnancies T: full term (37-40 wks) P: preterm (20-36 wks) A: Abortions/miscarriages (before 20 ws) L: Living children

27-year-old woman presents to the office due to new vaginal discharge. She has a long-standing history of recurrent vaginal yeast infections, for which she uses fluconazole 150 mg. She took two doses last week without relief. She reports having a thin, homogenous vaginal discharge that smells fishy. Which of the following organisms contributes to the suspected finding? Candida glabrata Gardnerella vaginalis Treponema pallidum Trichomonas vaginalis

Gardnerella vaginalis vaginitis - tx w metronidazole 500mg POx2 for 7 days

25-year-old G2P1 woman at 33 weeks gestation presents to the emergency department after feeling a gush of vaginal fluid 2 hours ago. On speculum examination, clear fluid is seen pouring out of the cervical os and pooling into the vaginal vault. Nitrazine testing reveals a pH of 7.2, and microscopic examination demonstrates a ferning pattern. Ultrasound is significant for oligohydramnios. Which of the following is the most common risk factor for the patient's condition? Antepartum bleeding Cigarette smoking Gardnerella vaginalis infection Increased body mass index

Gardnerella vaginalis infection Genital tract infection is *MOST COMMON* rf for preterm prelabor rupture of membranes (PPROM)

A 16-year-old high school gymnast presents to her gynecologist with concerns about menstruation. She states that most of her friends started menstruating in middle school, and she is worried because she has never had a period. Her vital signs are normal, her height is in the 5th percentile, and her BMI is 20.1 kg/m2. She states that she has always been the shortest girl in her class. No abnormal weight changes are noted on her growth chart. The patient has appropriate secondary sexual characteristics, and her genitourinary examination is unremarkable. Physical examination is significant for low-set ears, low hairline, and a high arched palate. Urine pregnancy test is negative. Which of the following is the most common cause of the patient's condition? Excessive exercise Gonadal dysgenesis Müllerian agenesis Polycystic ovary syndrome

Gonadal dysgenesis (Turner syndrome) MCC of primary amenorrhea

Which sign associated with pregnancy is characterized by softening of the cervix? Chadwick sign Goodell sign Hegar sign Osiander sign

Goodell sign (softening of cervix at 4-6 wks) *chadwick - blue discoloration *Hegar- softening of uterus (upper part enlarged and lower part is empty) *Osiander - pulsation through lateral vagina fornices

32-year-old woman who is lactating presents to the office due to localized breast pain and malaise. Relevant findings on physical exam include a temperature of 100.9°F with a fluctuate, tender, erythematous mass in the upper outer breast quadrant. The patient has attempted to treat her symptoms with warm compresses, massage, and a course of antibiotics prescribed by her primary care provider. Which of the following results would a culture of aspirated fluid most likely have? Gram-negative rods Gram-positive cocci in clusters Gram-positive diplococci No antimicrobial growth

Gram-positive cocci in clusters Staph. Aureus

24-year-old pregnant woman presents to the obstetrics clinic for routine prenatal care at 36 weeks gestation. She is Rh negative. Which of the following prenatal diagnostic or therapeutic tests is appropriate at this visit? Anatomy survey ultrasound Group B Streptococcus screening Oral glucose tolerance test Rho(D) administration

Group B Streptococcus screening done 35-37 weeks US - 18-22 wks GCT - 24-28 wks Rhogam - 26-28 wks and after delivery if baby is Rh+

22-year old G1P1 woman undergoes a cesarean section at 37 weeks gestation because she is HIV positive and her RNA viral load is 1,500 copies/mL. She has a recent history of herpetic breast lesions 1 month ago that have resolved. She was successfully treated for tuberculosis 2 years ago. After a 5-year history of cocaine use, she quit upon becoming pregnant, and her current urine drug screen is negative. Case management has provided the patient with resources for obtaining affordable baby formula. Which of the following in the patient's history is a contraindication for breastfeeding? History of cocaine use History of tuberculosis HIV status Recent herpetic breast lesions

HIV status

19-year-old woman presents with a tender genital ulcer that she noticed 2 days ago. She also reports dysuria and dyspareunia. On exam, you note a genital ulcer that has a ragged border and purulent base. A palpable, tender left inguinal lymph node is also present. Which of the following organisms is the most likely causative organism? Chlamydia trachomatis Haemophilus ducreyi Phthirus pubis Treponema pallidum

Haemophilus ducreyi chancroid PAINFUL ulcer (chancre) with ragged boarder and purulent base with buboes (inguinal lymphadenopathy)

35-year-old woman undergoes a transvaginal ultrasound showing hypoechoic, round, and well-circumscribed uterine tumors. Which of the following is the most common symptom of the most likely diagnosis? Heavy menses Infertility Intermenstrual bleeding Pelvic pain

Heavy menses uterine fibroids mc sx is heavy or prolonged menses

24-year-old woman presents to the primary care provider with concerns about a sexually transmitted infection. She states she had oral sex with a new partner 1 week ago. The patient states that she has had a tingling and burning sensation near her vagina over the past 2 days. She reports no fever or vaginal discharge. Physical examination reveals the presence of vaginal lesions, seen in the image above. Which of the following is the causative agent of the most likely diagnosis? Chlamydia trachomatis Herpes simplex virus Human papillomavirus Neisseria gonorrhoeae

Herpes simplex virus grouped vesicles that progress into painful shallow ulcers

30-year-old woman is in labor and has just delivered the fetal head. The fetal head then retracts into the perineum and the rest of the fetus cannot be easily delivered with gentle and downward traction. Which of the following is the most important risk factor for shoulder dystocia? Diabetic pregnancies with normal fetal birth weight Female sex High birth weight in nondiabetic pregnancies Preterm birth

High birth weight in nondiabetic pregnancies turtle sign in shoulder dystocia

woman who closely monitors her menstrual cycles notices clear cervical mucus similar to raw egg white. Which of the following describes the estrogen and progesterone hormone levels at her most likely stage of the menstrual cycle? High estrogen and low progesterone High estrogen and progesterone Low estrogen and high progesterone Low estrogen and progesterone

High estrogen and low progesterone transition between follicular and luteal phase (ovulation)

50-year-old woman presents to the clinic for a routine well-woman exam. She reports it has been 12 months since her last menstrual period. Which of the following is the most common symptom of menopause? Hot flashes Mood symptoms Sleep disturbance Vaginal dryness

Hot flashes

28-year-old woman with previous regular menses presents with a complaint that she has not experienced menses in the last 3 months. She reports episodes of headaches, mood swings, nausea, and vomiting. A physical exam is unremarkable at this time. Which of the following is the most appropriate initial diagnostic study? Follicle-stimulating hormone Human chorionic gonadotropin hormone Luteinizing hormone Thyroid-stimulating hormone

Human chorionic gonadotropin hormone

46-year-old woman presents to her gynecology office to discuss her colposcopy results. She was told the results were significant for carcinoma, and she needs a total abdominal hysterectomy with a bilateral salpingo-oophorectomy. What is the greatest risk factor for this patient's diagnosis? Diethylstilbestrol exposure Early-onset menarche Human papillomavirus Nulliparity

Human papillomavirus cervical carcinoma RF

34-year-old woman presents to the office with new vaginal symptoms. She noticed vulvar and vaginal pruritus 1 week ago. She developed a thick, white vaginal discharge that resembled curds 3 days later. She is concerned about a possible drug reaction since symptoms began after finishing a 5-day course of azithromycin for a sinus infection. What finding on a potassium hydroxide wet mount will confirm your diagnosis? Clue cells Epithelial cells Hyphae and spores Trichomonads

Hyphae and spores candidiasis

34-year-old woman who is 8 weeks pregnant presents for a routine prenatal examination and reports persistent vomiting up to six times a day, and she has been unable to tolerate anything by mouth for the past 2 days. Her weight today is 114 pounds compared to 120 pounds 2 weeks ago. A blood test is ordered. Which of the following electrolyte imbalances is associated with the most likely diagnosis? Hyperchloremia Hypokalemia Hyponatremia Hypophosphatemia

Hypokalemia

A 38-year-old woman presents along with her 42-year-old husband to discuss difficulties conceiving. They have had consistent, unprotected intercourse for 8 months without a positive pregnancy test. She has never been pregnant, and he has never fathered a child. She has a history of regular menses occurring every 30 days. They both report no history of sexually transmitted infections. Neither individual has a family history of infertility or genetic diseases. Neither of them smoke or use illicit drugs, although they have an occasional drink with dinner. Which diagnostic study is most appropriate, according to this history? Hysterosalpingography and scrotal ultrasound Hysteroscopy and semen analysis Scrotal ultrasound and semen analysis Semen analysis

Hysteroscopy and semen analysis

32-year-old G2P2 woman presents to the office due to left breast tenderness. She is 2 months postpatrum and had a normal vaginal delivery of a healthy baby girl. Her infant is entirely breastfed and has been gaining weight appropriately. Other than left breast tenderness and some redness that she noticed shortly before the appointment, she feels well. She has a history of mastitis with her first pregnancy. Her vital signs show a blood pressure of 119/77 mm Hg, pulse of 88 bpm, temperature of 98.6°F, and respirations of 10 breaths per minute. On physical exam, her left breast is engorged and has a 1.0 cm area of erythema and tenderness on the lateral aspect of the breast. There is no edema or dimpling of the skin. Milk is easily expressed from the left side. Which of the following is the most appropriate initial management for this patient while she continues to completely empty the involved breast? Dicloxacillin 500

Ibuprofen 800 mg PO as needed (dicloxacillin 500mg PO for more severe symptoms) lactational mastitis

14-year-old girl presents to the clinic with abnormal uterine bleeding over the previous 6 months. Medical history reveals that menarche occurred 1 year ago. The patient states that she has been having menstrual bleeding at sporadic intervals greater than 28 days apart. Vital signs show BP 120/72 mm Hg, HR 77 bpm, RR 16/min, and BMI 21 kg/m2. A urine pregnancy test is negative. Which of the following is the most likely cause of this patient's abnormal uterine bleeding? Adenomyosis Hyperthyroidism Immature hypothalamic-pituitary axis Polycystic ovary disease

Immature hypothalamic-pituitary axis MCC of AUB in adolescent within first 1-2 years of menarche

25-year-old G1P0 woman at 20 weeks gestation presents to her obstetrician for a routine prenatal visit. She is concerned because her hemoglobin is 11.0 g/dL, which is lower than normal for her. Which of the following physiologic changes of pregnancy is most likely responsible for the patient's low hemoglobin level? Decrease in red blood cell mass Decrease in responsiveness to norepinephrine Increase in glomerular filtration rate Increase in plasma volume

Increase in plasma volume dilution anemia

50-year-old woman presents to the gynecologist to discuss her mammogram results. A firm, notender, irregular mass was palpated in the upper outer region of her left breast, and a mammogram identified a lesion suspicious for invasive breast cancer. Which of the following is the most common type of invasive breast cancer? Infiltrating ductal carcinoma Infiltrating lobular carcinoma Medullary carcinoma Tubular carcinoma

Infiltrating ductal carcinoma MC in situ (invasive) breast carcinoma

60-year-old woman presents to the clinic reporting a rapidly enlarging, tender mass in her left breast. She was treated with antibiotics 3 weeks ago when she first noticed the mass, but her symptoms did not abate. She is afebrile, and other vital signs are within normal limits. On exam, an area of erythema involving the entire inferior half of the left breast is noted. The area is tender to palpation, with a firm mass underlying the erythema, and the skin of the breast is edematous, causing the pores of the skin to be very pronounced. There is a palpable, immobile, nontender left axillary node as well. Which of the following is the most likely diagnosis? Breast abscess Erythema nodosum Inflammatory breast cancer Paget disease of the breast

Inflammatory breast cancer

32-year-old woman returns to her obstetrician for a follow-up of her glucose tolerance testing results and is advised her blood glucose levels were elevated with each of the tests. She has been trying to adjust her diet ever since her first prenatal visit and is frustrated that her glucose levels are elevated. She has obesity at baseline and a family history of diabetes mellitus. Her interim history and exam are otherwise unremarkable. Based on her test results, you discuss that medical management is recommended. Which of the following is the first-line pharmacologic therapy recommended for this patient? Glyburide Insulin Metformin Myoinositol

Insulin considered first-line therapy

57-year-old woman presents for a follow-up to a recent abnormal breast exam. She was noted to have a nontender, hard, fixed breast lump with associated enlarged axillary lymph nodes on her annual breast exam. Her mammogram was also significant for a spiculated mass highly suspicious for malignancy. What is the most common form of this patient's diagnosis? Inflammatory breast carcinoma Invasive ductal carcinoma Invasive lobular carcinoma Paget carcinoma

Invasive ductal carcinoma MC type of breast cancer

52-year-old woman presents to her gynecologist with reports of vaginal pressure, dyspareunia, urinary retention, and the sensation that something is falling out of her vagina. Her urinalysis is unremarkable. Speculum examination reveals an anterior vaginal wall prolapse. Which of the following physical examination findings is associated with a risk factor for the most likely diagnosis? Chandelier sign Joint hypermobility Scar from a previous tube thoracostomy Thin body habitus

Joint hypermobility (associated with certain connective tissue disorders - Ehlers-danlos syndrome) cystocele

A 63-year-old woman presents to the gynecology clinic for a routine wellness exam. Physical examination reveals a bulging of the apex of the vaginal vault. Which of the following is recommended to prevent this condition? Abstinence Kegel exercises Smoking cessation Weight lifting

Kegel exercises pelvic organ prolapse

28-year-old woman who is 28 weeks pregnant presents to the emergency department after she was involved in a motor vehicle collision < 1 hour ago. She immediately developed abdominal cramping and scant vaginal bleeding. This is her first child, and she has an Rh-negative blood type. The baby's father was recently tested and found to be Rh-positive. Which of the following tests will give a quantitative measurement of fetal red blood cells in the maternal blood? Direct Coombs test Kirby-Bauer test Kleihauer-Betke test Rosette test

Kleihauer-Betke test *quantitative* measurement of amount of fetal blood cells in maternal blood (Rosette test - initial test *qualitative* test)

A 30-year-old nulliparous woman presents with dysmenorrhea, dyschezia, and dyspareunia. On exam, you note a retroverted uterus and a palpable adnexal mass. Which of the following is needed for the definitive diagnosis of the suspected disorder? Abdominal ultrasound Laparoscopy and biopsy Pelvic MRI Serum cancer antigen 125

Laparoscopy and biopsy endometriosis

26-year-old G1P0 woman is in the first stage of labor. Which of the following accurately describes the fetal heart pattern shown above? Accelerations Early decelerations Late decelerations Variable decelerations

Late decelerations

27-year-old G1P0 woman at 38 weeks gestation presents to her obstetrician for a routine checkup. Which of the following should the obstetrician use to assess the position of the fetus? Gaskin maneuver Leopold maneuver McRoberts maneuver Zavanelli maneuver

Leopold maneuver (determine fetal lie, presentation, and position) Gaskin - all fours to reduce shoulder dystocia Mcroberts - flexion of legs to abdomen with suprapubic pressure to reduce shoulder dystocia Zavanelli - last resort placement of fetal head back into pelvis to prepare for c-section

A 36-year-old woman presents to her gynecologist with complaints of heavy menstrual cycles and dysmenorrhea for the past 5 months. She states her periods have previously been normal. She reports no dyspareunia or dyschezia. Pelvic examination reveals the presence of an enlarged, mobile, globular, boggy uterus with no adnexal tenderness. No cervical discharge is seen on speculum exam. Her urine pregnancy test is negative. Transvaginal ultrasound is significant for an enlarged uterus. Which of the following treatments is most appropriate for the suspected diagnosis? Danazol Estrogen-progestin contraceptive Levonorgestrel-releasing intrauterine device Mifepristone

Levonorgestrel-releasing intrauterine device preferred tx in adenomyosis

26-year-old G1P0 woman at 28 weeks gestation by last menstrual period presents for her first prenatal visit. She is unsure of her vaccination status. Which of the following vaccines is contraindicated in this patient? Hepatitis A vaccine Live attenuated influenza vaccine Meningococcal meningitis vaccine

Live attenuated influenza vaccine (Live vaccines contraindicated: MMR, Varicella, Oral polio, BCG and live zoster vaccine) (hpv not recommended)

25-year-old woman presents to labor and delivery at 38 weeks gestational age with contractions. She reports she has had contractions every 5 minutes for the past 2 hours. Cervical examination reveals she is dilated to 4 cm. The patient is requesting pain medication. Which of the following is a relative contraindication to neuraxial analgesia? Aortic regurgitation Fever Local infection at the site of neuraxial analgesia puncture Preeclampsia

Local infection at the site of neuraxial analgesia puncture elative contraindications include coagulopathy, infection at the site of neuraxial analgesia puncture, and increased intracranial pressure.

woman reports to the office 2 days postpartum due to persistent bloody discharge. She describes the bleeding as red-brown and states it resembles her menstrual discharge. Which of the following is the correct name and appropriate duration of this discharge? Lochia alba, which may continue for up to 8 weeks Lochia rubra, which should subside in a few days Lochia serosa, which should subside in a few days Lochia serosa, which should subside in a few weeks

Lochia rubra, which should subside in a few days followed by loch serosa (pinkish brown coloring lasting 2-3 weeks), then loch alba (yellowish-white discharge up to 8 weeks)

A 40-year-old woman presents to the clinic with concerns for a right breast lump. Physical exam reveals a 3 cm mass located in the right upper quadrant of the right breast. Mammogram reveals a 3 cm by 1.5 cm spiculated mass with notable microcalcifications. A core needle biopsy is also obtained and shows well-differentiated tumor cells that infiltrate the stroma suggestive of a stage I infiltrating ductal carcinoma. Immunohistochemical staining reveals that the mass is estrogen and progesterone receptor positive. What is the next most appropriate step in management? Chemotherapy Hormone therapy Lumpectomy with sentinel lymph node biopsy Mastectomy with removal of the axillary lymph nodes

Lumpectomy with sentinel lymph node biopsy

Which of the following is the best patient education regarding prenatal care to give to a pregnant woman with an uncomplicated pregnancy? Do not engage in sexual intercourse Do not wear a seatbelt Maintain hydration and regularly move your lower extremities during air travel Sleep in a supine position during the third trimester

Maintain hydration and regularly move your lower extremities during air travel inc risk of thromboembolism

22-year-old G1P1 woman presents to her clinician for a 6-week postnatal follow-up. She states her right breast has been swollen, warm, and tender for 3 days. She reports she has been having trouble breastfeeding. Her vital signs are T 100.6°F, BP 132/68 mm Hg, HR 82 bpm, RR 18 breaths per minute, and pulse oximetry 99% on room air. Physical examination reveals a central indurated area of the right breast. It is tender to palpation, firm, warm, swollen, and erythematous without fluctuance. Which of the following is the most likely diagnosis? Breast abscess Fibrocystic breast disease Inflammatory breast cancer Mastitis

Mastitis lactating within 3 months of delivery *Dicloxacillin*, cephalexin, clindamycin (staph and strep) Trimethoprim-sulfamethoxazole (MRSA)

25-year-old woman presents to the emergency department 8 weeks postpartum complaining of right-sided breast pain and malaise. She has been lactating since delivery. Vital signs are T of 101.3°F, HR 114 of bpm, BP of 110/75 mm Hg, RR of 20/min, and oxygen saturation of 98% on room air. Physical exam reveals a palpable, tender, and fluctuant nodule in the right breast surrounded by erythema. Which of the following is the most likely diagnosis? Inflammatory breast cancer Mastitis with an abscess Mastitis without an abscess Plugged duct

Mastitis with an abscess

33-year-old G1P0 woman at 43 weeks gestation presents to the labor and delivery unit for induction of labor. Which of the following physical characteristics is most likely to be seen in her infant? Increased lanugo hair Increased vernix caseosa Meconium staining Moist, thick skin over the thighs and buttocks

Meconium staining

36-year-old woman who is 37 weeks pregnant with her first child presents to the hospital after feeling a gush of liquid from her vaginal area and seeing clear fluid while in the shower about 2 hours ago. She was unsure what it was but has been feeling more pelvic pressure and decided to come in for an evaluation. She is afebrile and has not had any contractions. A speculum examination reveals leakage from the cervical os with pooling of fluid in the vaginal vault. Which of the following is the best next step in management for this patient? Administer a tocolytic Expectant management and delivery Immediate cesarean section Medical induction with oxytocin

Medical induction with oxytocin for prompt delivery Prelabor rupture of membranes (PROM) - more than 37 weeks gestation with membrane rupture before labor

35-year-old G2P1 woman at 41 weeks gestation is in a prolonged second stage of labor. Delivery of the fetus is blocked by perineal tissue. A category III tracing is unresponsive to resuscitative measures, and the obstetrician decides to perform an episiotomy. Which of the following is the preferred type of episiotomy to minimize the risk of anal sphincter laceration? Anterior episiotomy J incision Lateral episiotomy Mediolateral episiotomy

Mediolateral episiotomy

A 32-year-old woman presents with a complaint of increased menstrual bleeding for the past several months. She reports that her periods used to be moderate flow for 2-3 days. However, now they last for 7-10 days, and she is changing her pad or tampon every hour. Her pelvic exam is unremarkable, and no blood is seen at this time. Endometrial biopsy shows hyperplasia with atypia. A dilation and curettage reveals no evidence of endometrial carcinoma. The patient is very concerned because she has been trying to get pregnant. Which of the following is recommended for this patient? Anastrozole Hysterectomy Megestrol acetate Observation

Megestrol acetate (or depot medroxyprogesterone acetate) treat with progestin therapy to preserve fertility

22-year-old woman presents with a complaint of painless heavy vaginal bleeding. She reports that her menstrual cycle occurs at irregular intervals and usually lasts more than 35 days. A physical exam is unremarkable at this time and laboratory studies are within normal limits. Which of the following is the most likely diagnosis? Menometrorrhagia Menorrhagia Metrorrhagia Polymenorrhea

Menometrorrhagia heavy, irregular vaginal bleeding

23-year-old woman presents to the office complaining of lower abdominal pain for the last 2 days. The pain is worse on the left, and she cannot seem to find a comfortable position. Her last menstrual period was 2 months ago, and her menses are usually every 28 days. She reports very light spotting on day 32 of her cycle. Her vital signs show a blood pressure of 121/78 mm Hg, pulse of 94 bpm, temperature of 98.6°F, and respirations of 10 breaths per minute. An abdominal exam reveals tenderness in the left lower quadrant. Bimanual exam reveals a 6 cm uterus with smooth contours. There is adnexal fullness on the left. The cervix is closed and without abnormalities. A urine human chorionic gonadotropin is positive, and serum value is 2,000 mlU/L. A transvaginal ultrasound reveals a left adnexal mass of 1.8 cm with free fluid in the cul-de-sac. What is the most appropriate management option at this time? Intravenous flui

Methotrexate ectopic pregnancy

35-year-old woman presents to the clinic with white and thin vaginal discharge. Physical examination reveals white homogeneous vaginal discharge with a pH of 5. There is no vulvar or vaginal erythema or edema on exam. You also notice a fishy odor when 10% potassium hydroxide is added to a sample of the vaginal discharge. Which of the following is the best treatment for the most likely diagnosis? Azithromycin Ceftriaxone Doxycycline Metronidazole

Metronidazole Bacterial Vaginosis

29-year-old woman who is 25 weeks pregnant presents to the office due to increased foul-smelling vaginal discharge over the past week. A physical exam reveals the presence of runny and milky discharge that has a fishy odor. The pH of the vaginal sample is 5.0, and there are clue cells identified on wet mount microscopy. Which of the following is a first-line treatment for this condition? Metronidazole 2 g one dose orally Metronidazole 250 mg orally three times daily for 7 days Tinidazole 1 g orally once daily for 5 days Tinidazole 2 g one dose orally

Metronidazole 250 mg orally three times daily for 7 days BV tx: metronidazole 500mg 2x or 250 3x daily for 7 days *metronidazole 2g one dose --> treat trich* Tinidazole 1 g orally once daily for 5 days (c) - for non pregnant BV

A woman presents to the breast clinic with a new unilateral right-sided breast lesion. Physical examination reveals a well-circumscribed, nontender, and smooth 2.5 cm mass. Ultrasound reveals a solid and well-circumcised lesion. Which of the following additional findings is consistent with the most likely diagnosis? Axillary lymphadenopathy Lesion occurring in a 55-year-old patient Mobile quality to the lesion Nipple discharge

Mobile quality to the lesion Fibroadenomas

36-year-old woman is 13 weeks pregnant with twins. There are two distinct fetuses and the presence of a T sign on ultrasound. Which of the following most likely describes this patient's twin gestation? Dichorionic, diamniotic gestation Dichorionic, monoamniotic gestation Monochorionic, diamniotic gestation Monochorionic, monoamniotic gestation

Monochorionic, diamniotic gestation T-sign --> one placenta, two separate amniotic sacs

15-year-old girl presents to her gynecologist with concerns about amenorrhea. She states all of her friends have started menstruating, but she has not. Physical examination reveals normal breast development, axillary hair, and pubic hair. The labia and hymen are normal in appearance. Pelvic examination reveals the presence of a small vaginal pouch. Transrectal ultrasound reveals the presence of normal-appearing ovaries bilaterally without the presence of a uterus. Serum beta-human chorionic gonadotropin is < 1 mIU/mL. What is the most likely diagnosis? Müllerian agenesis Pregnancy Primary ovarian insufficiency Turner syndrome

Müllerian agenesis congenital absence of the vagina with variable uterine development of agenesis sparing the ovaries

28-year-old woman presents to the clinic reporting amenorrhea. She states her last period was 6 weeks ago. Vital signs are within normal limits, and urinalysis is negative. Which of the following is most consistent with a diagnosis of pregnancy? Dysuria Headache Lower abdominal pain Nausea

Nausea

35-year-old woman, who is currently breastfeeding, presents with left breast tenderness, redness, and warmth. On exam, you note a fluctuant, tender, and palpable left breast mass without ischemia or necrosis. Breast ultrasound demonstrates a fluid collection within the left breast. Which of the following is the most appropriate clinical intervention? Needle aspiration, antibiotics, and continuing breastfeeding Needle aspiration, antibiotics, and discontinuing breastfeeding Surgical drainage, antibiotics, and continuing breastfeeding Surgical drainage, antibiotics, and discontinuing breastfeeding

Needle aspiration, antibiotics, and continuing breastfeeding

35-year-old woman presents with complaints of vaginal itching and purulent discharge for the last 3 days. She reports that it started after having intercourse with a new partner. A cervical swab is positive for gram-negative diplococci. Which of the following is the most likely cause? Chlamydia trachomatis Neisseria gonorrhoeae Treponema pallidum Trichomonas vaginalis

Neisseria gonorrhoeae

Which of the following fetal abnormalities is associated with an elevated alpha-fetoprotein level? Aneuploidy Cystic fibrosis Neural tube defects Tay-Sachs disease

Neural tube defects

27-year-old G2P1001 woman presents to labor and delivery at 33 weeks and 2 days of gestation with regular uterine contractions for 2 hours. She reports no vaginal bleeding or clear leakage of fluid. Pelvic examination reveals cervical dilation to 3 cm. Which of the following is the best recommendation for tocolytic therapy? Indomethacin Nifedipine Terbutaline Tocolytic therapy is not indicated

Nifedipine CCB - nifedipine first-line agent for tocolytic therapy between 32-34 weeks (second-line between 24-32; indomethacin first-line)

22-year-old woman presents to the clinic with vaginal discharge a week after having unprotected intercourse with a new sexual partner. Which of the following is considered the best diagnostic test for the most common bacterial sexually transmitted infection? Antigen detection Culture of a vaginal swab Genetic probe testing Nucleic acid amplification testing

Nucleic acid amplification testing test of choice

21-year-old woman presents to the office for lower abdominal pain and cramping for several days. She states that she had typical cramping and pain with her period but then continued to have pain even after menstruating. Her pain is across the lower abdomen and is constant. It is worse during intercourse, and she has been bleeding thereafter. She has multiple sexual partners and says that she does not always use barrier protection. A physical exam reveals some mucopurulent cervical discharge, and there is cervical motion tenderness on a bimanual exam. Which of the following studies should be ordered to help diagnose this condition? Endometrial sampling Nucleic acid amplification tests Papanicolaou smear Potassium hydroxide microscopy

Nucleic acid amplification tests *Test for Chlamdia, gonorrhea, and mycoplasma genitalium PID

41-year-old G1P0 woman presents for a routine prenatal visit. She is 35 weeks pregnant. She reports a normal pregnancy course to date and feels well today. She has experienced a 25 lb weight gain over the last 8 months and reports recent lower extremity swelling. The swelling usually resolves when she elevates her feet at night but not always. Her bowel movements have been normal, but she reports frequent urination and lower abdominal pain. The pain is occasional, sharp, and fleeting. It usually occurs after sudden movements. She also reports visual flashes of light, which are infrequent but occur daily. She reports neither eye pain nor blurry vision. Upon arrival, her vital signs show a blood pressure of 134/92 mm Hg, pulse of 80 bpm, temperature of 98.6°F, and respirations of 12 breaths per minute. Pupils are equal, round, and reactive to light and accommodation bilaterally. Cardiovascular exam reveals a regular r

Occasional flashes of light in her vision signs of preeclampsia

A 28-year-old woman presents to the emergency department with acute left-sided pelvic pain and vomiting. Physical examination reveals left-sided lower abdominal tenderness. Urine pregnancy test is negative. The transvaginal ultrasound reveals an enlarged and rounded left ovary with decreased Doppler flow to the left ovary. Which of the following is the most likely diagnosis? Ectopic pregnancy Ovarian cyst rupture Ovarian torsion Pelvic inflammatory disease

Ovarian torsion

28-year-old woman presents to her gynecologist with concerns about pain during sexual intercourse. She reports no fever, vaginal discharge, or vaginal odor but has dull, crampy pain with menstruation and bowel movements. Pelvic examination reveals a fixed and retroverted uterus, no cervical motion tenderness, and no adnexal masses. Which of the following is the most commonly affected site of the patient's condition? Cervix Fallopian tubes Ovaries Uterus

Ovaries MC site of endometriosis

58-year-old woman presents to her primary care provider with complaints of urinary incontinence. She states that, sometimes, she cannot make it to the bathroom in time and wets herself. On speculum examination, the patient is asked to cough, which results in the leakage of a small amount of urine. Her urinalysis is unremarkable. She is diagnosed with mixed urinary incontinence. Which of the following treats urge incontinence rather than stress incontinence? Kegel exercises Oxybutynin Pessary placement Topical estrogen

Oxybutynin antimuscarinic

70-year-old woman presents to her gynecologist with concerns about a pruritic vaginal lesion. She was prescribed a vaginal cream for eczema by her primary care provider 6 weeks ago, but her symptoms have not improved. Physical examination is significant for a well-demarcated, slightly raised, erythematous vulvar lesion with an eczematoid appearance, as shown above. The rest of her physical examination is unremarkable. Which of the following is the most likely diagnosis? atrophic vaginitis bartholian gland cyst lichen planus Paget disease of the vulva

Paget disease of the vulva

22-year-old woman presents to the women's health clinic with genital sores and dysuria. Vital signs are T of 99.5°F, HR of 94 bpm, BP of 135/88 mm Hg, RR of 20/min, and oxygen saturation of 98% on room air. Which of the following additional findings on genital exam would be most suggestive of genital herpes simplex virus infection? Painful and fluid-filled vesicles Painless ulceration with raised and indurated margins Skin-colored cauliflower-shaped lesions Vaginal discharge

Painful and fluid-filled vesicles

22-year-old woman in the Caribbean presents to the public health clinic with buboes in her right inguinal area. She had unprotected intercourse with a new sexual partner 6 weeks ago. Which of the following clinical manifestations is most commonly seen in primary infection with this condition? Painful genital ulcer Painless genital ulcer Rectal discharge Tenesmus

Painless genital ulcer lymphogranuloma venereum (from chlamydia)

25-year-old G1P0 woman presents to a routine prenatal appointment at 32 weeks gestation. She has a condition in which the placental tissue extends over the internal cervical os. Which of the following is the most common symptom of this condition during the second half of pregnancy to educate the patient about? Leakage of clear fluid Painful vaginal bleeding Painless vaginal bleeding Uterine contractions

Painless vaginal bleeding placenta previa

27-year-old nulliparous woman presents to her gynecologist for a routine examination. She voices no concerns at this time. She is sexually active with three male partners and does not use any form of contraception. Her Pap smear report returns atypical squamous cells, cannot exclude high-grade squamous intraepithelial lesion. During colposcopy, a white, sharply demarcated lesion on the cervix is visualized after acetic acid is applied. Colposcopy is adequate, and cervical biopsy results reveal cervical intraepithelial neoplasia grade 2. The patient states she would like to have children in the future and the risks of treatment outweigh the risk of cancer. Which of the following is the best plan for management? Loop electrosurgical excision procedure Pap smear and colposcopy in 12 months Pap smear and colposcopy in 6 months Pap smear in 6 months

Pap smear and colposcopy in 6 months

32-year-old woman at 37 weeks gestation presents to the hospital with a persistent, severe headache, epigastric pain, and blurred vision. Vital signs indicate HR 96 bpm, RR 16 breaths per minute, BP 162/96 mm Hg, T 98.9°F, and oxygen saturation 98% on room air. While indicated lab results are pending, the patient is given antihypertensives and is started on the appropriate first-line medication for prophylactic seizure prevention. Which of the following can be used to assess the therapeutic level for this agent and monitor for drug toxicity? Blood pressure values Capillary refill Patellar reflex Pupillary response

Patellar reflex preeclampsia with end-organ involvement

A 64-year-old woman presents with concerning breast symptoms. She reports the symptoms began nearly 2 years ago when she noticed a left-sided breast mass. Since then, she noticed her left nipple appears to be smaller than the right and she has occasional yellowish nipple discharge from the left side. On physical exam, she has a fixed, poorly defined 0.8 cm mass on the left breast. There is overlying skin erythema with an orange peel appearance, as seen in the image above. You are able to express 0.5 cc of milky-yellow fluid from the nipple, although it is somewhat retracted. She also has left-sided axillary lymphadenopathy. A mammogram is ordered. Which physical exam finding is the most concerning for inflammatory breast neoplasia? Nipple retraction Peau d'orange skin changes of the breast Right upper quadrant breast mass Serous nipple discharge

Peau d'orange skin changes of the breast Inflammatory breast cancer

A 45-year-old woman presents to the office complaining of a new vulvar lesion. She reports the lesion first appeared 2 weeks ago and has become slightly larger. It is not tender and does not bleed. Her past medical history is significant for anxiety, depression, and HIV. She has no drug allergies. Her most recent CD4+ count was 750 cells/microL. She has a history of risky sexual behavior and reports four new sexual partners in the last 2 months. She prefers not to use condoms. On physical exam, there is a 1.1 cm shallow, painless, erythematous ulceration on the mucosal surface of her left labia. Serology testing, including a CD4+ count, is ordered, and results are pending. In the meantime, what is the most appropriate pharmacotherapeutic option for this patient? Doxycycline 100 mg orally for 14 days Imiquimod 1 packet tid for 16 weeks Penicillin 1 dose of 2.4 million units Valacyclovir 1 g bid for 10 days

Penicillin 1 dose of 2.4 million units syphilis

30-year-old G4P3003 woman who is 35 weeks pregnant arrives at the emergency department with severe abdominal pain and persistent vaginal bleeding that began approximately 2 hours ago. Vital signs are HR 105 bpm, BP 116/68 mm Hg, and RR 16 breaths per minute. Fetal heart monitoring shows late decelerations. An ultrasound is obtained, which is shown above. Which of the following is the most appropriate next step in management? Administer intravenous magnesium sulfate Administer intravenous steroids Administer intravenous tocolytics Perform cesarean section

Perform cesarean section placental abruption triad: severe abdominal pain, fetal distress, sudden onset of 3rd try bleeding

67-year-old G6P5 woman presents to the clinic complaining of pelvic pressure that has progressively worsened for months. She also complains she cannot empty her bladder completely. She is currently sexually active. Physical examination reveals collapse of the anterior vaginal wall. Which of the following is the recommended initial treatment? Anterior colporrhaphy Heavy resistance-based exercise Obliterative surgery Pessary

Pessary removable devices to support pelvic organs conservative(initial) tx: wt loss, Kegel exercise, pessaries

Which of the following skin surgical incisions is the preferred approach for most cesarean section deliveries? Joel-Cohen Munro Kerr Pfannenstiel Vertical midline

Pfannenstiel 2-3cm above symphysis pubis in curved shape (munro kerr - low transverse UTERINE incision)

24-year-old G1P0000 woman presents to the clinic at 36 weeks gestational age for a routine prenatal visit. Her blood pressure is noted to be 142/102 mm Hg and is 146/104 mm Hg on repeat measurement 4 hours later. She has no prior history of hypertension. Which of the following additional findings would support a diagnosis of severe preeclampsia? 200 mg of protein on a 24-hour protein collection Mild headache that resolves with acetaminophen Photopsias Subsequent repeat blood pressure of 159/109 mm Hg

Photopsias (floaters) vision changes = severe preeclampsia (>300 mg/24hr collection)

24-year-old woman presents to the emergency department after delivering a baby at a birthing center about 1 hour ago. She is pale and fatigued, with a blood pressure of 92/54 mm Hg and a heart rate of 124 beats per minute. She is wearing a pad that she states has been changed multiple times within the past hour and has been filled with bright red blood and clots. Which of the following placental complications has the greatest risk for postpartum hemorrhage due to the depth of invasion into the uterine myometrium? Placenta accreta Placenta increta Placenta percreta Placenta previa

Placenta percreta previa (covering cervix) accreta (invade uterine wall - general complication, least severe) increta (deeper invasion, less severe) percreta (most severe - greatest risk of PPH)

A 32-year-old G3P2 woman, who actively smokes tobacco, presents to her obstetrics office reporting bright red vaginal bleeding that she first noticed this morning. She is currently 29 weeks gestation. As per the patient, she is not having any abdominal or pelvic pain or any cramping sensation. On exam, there is no evidence of fetal distress, and there is no evidence the patient is in active labor. She reports no recent trauma. Which of the following is most likely the patient's diagnosis? Placenta previa Placental abruption Spontaneous abortion Vasa previa

Placenta previa painless bright red bleeding

23-year-old woman presents to her gynecologist for routine prenatal care. She is at 35 weeks gestation with her first pregnancy. She reports ongoing epigastric pain, which she associates with reflux, and in the last week, she has had a persistent headache. Her vital signs are concerning for a blood pressure of 150/100 mm Hg. Her urine dipstick is negative for protein. She is advised to return the next day for repeat evaluation, including vital signs and additional laboratory tests. On her return visit, her blood pressure is unchanged. Which of the following laboratory findings confirms the diagnosis? Creatine concentration > 0.8 mg/dL Platelets < 100,000/µL Proteinuria > 100 mg in 24 hours Proteinuria on urine dipstick

Platelets < 100,000/µL bp > 140/90 mm Hg on two occasions at least 4 hours apart, or 160/110 mm Hg on one occasion without severe features must also have proteinuria > 300 mg over 24 hours or a urine protein to creatinine ratio > 0.3 mg/dL WITH severe features: at least one additional symptom or laboratory finding indicative of end-organ dysfunction, namely pulmonary edema, new-onset cerebral or visual disturbances, severe and persistent right upper quadrant or epigastric pain, thrombocytopenia (platelets < 100,000/µL), progressive kidney insufficiency (creatinine concentration > 1.1 mg/dL), or serum transaminase two or more times the upper limit of normal

24-year-old G2P1 woman at 24 weeks gestation presents to her obstetrician for a routine visit. Her blood glucose for the 50 gram 1-hour glucose challenge test is 185 mg/dL. A 100 gram 3-hour glucose tolerance test is ordered, and her blood glucose is 110 mg/dL fasting, 205 mg/dL at 1 hour, 180 mg/dL at 2 hours, and 165 mg/dL at 3 hours. Her glucose challenge test was negative during her last pregnancy. Which of the following complications is associated with the patient's current condition? Aplastic anemia Neonatal hyperglycemia Polyhydramnios Small for gestational age

Polyhydramnios

24-year-old woman at 37 weeks gestation presents to the office for a routine obstetrics exam. An ultrasound reveals an amniotic fluid index of 4 cm and no signs of fetal abnormalities. Which of the following is a risk factor for this condition? Fetal esophageal atresia Fetal hyperglycemia Preeclampsia Trisomy 21

Preeclampsia oligohydramnios (lower than normal volume) - AFI <5cm or deepest pocket <2cm MCC: rupture of membranes Other cases: dec. kidney fx, HTN/preeclampsia (placental insufficiency) *other choices are RF for polyhydramnios*

34-year-old woman who is at 33 weeks gestation returns to the office for a routine obstetric appointment. On physical examination, she has a blood pressure of 165/84 mm Hg and has evidence of proteinuria with a reading of 3+ found on urine dipstick test. She has never had a history of hypertension. Laboratory tests reveal a platelet count of 90,000 /μL and a creatinine of 1.6 mg/dL. She reports a mild headache that began a few days ago that has not been relieved with over-the-counter medications. A physical exam and fetal ultrasound are unremarkable. Which of the following is the most likely diagnosis? Eclampsia Hemolysis, elevated liver enzymes, and low platelet count syndrome Preeclampsia with severe features Preeclampsia without severe features

Preeclampsia with severe features

woman presents to the clinic for evaluation of a left breast mass located at the 9 o'clock position approximately 3 cm from her areola. The mass is mobile, round, hard, and has discrete borders. The patient states that she found the mass incidentally several months ago, and it has not changed in size. In what stage of female reproduction is this benign mass most likely to be found? Perimenopause Postmenopause Premenarche Premenopause

Premenopause fibroadenoma

A 20-year-old woman presents to her primary care provider in January with recurrent episodes of increased irritability, anxiety, and sadness for the past 3 months. She states the holidays are a stressful time for her because she does not get along well with her family. Upon further questioning, she states her mood seems normal most of the time, then all of a sudden, her symptoms will begin and last for almost a week. Which of the following is the most likely diagnosis? General anxiety disorder Premenstrual dysphoric disorder Premenstrual syndrome Seasonal affective disorder

Premenstrual syndrome (PMS) (PMD - more severe *5 symptoms* hinder life, most of preceding year)

33-year-old G2P1 woman presents to the emergency department reporting severe abdominal pain and dark red vaginal bleeding. She is currently 37 weeks pregnant and appears to be in labor. On exam, the baby is bradycardic, and a bedside ultrasound shows separation of the placenta edge. What is the greatest risk factor for this patient's condition? Abdominal trauma Cocaine use Eclampsia Previous placental abruption

Previous placental abruption rest are RF but not greatest or MC

Which hormone is responsible for uterine ripening to allow proper implantation of a fertilized ovum? Estrogen Follicle-stimulating hormone Luteinizing hormone Progesterone

Progesterone

30-year-old nulliparous woman presents to her gynecologist with prolonged and heavy menstrual bleeding for the past 6 weeks. Upon further questioning, she states she has also had abdominal discomfort. She is not currently taking any medications. She is sexually active with three male partners and does not use contraception. Pelvic examination is significant for an irregular, enlarged uterus without cervical motion tenderness or adnexal masses. Transvaginal ultrasound confirms the suspected diagnosis. Which of the following treatments is most appropriate for the patient at this time? Azithromycin only Ceftriaxone and azithromycin Gonadotropin-releasing hormone agonist Progestin-releasing intrauterine device

Progestin-releasing intrauterine device leiomyomas (uterine fibroids)

Which of the following hormones is responsible for breast milk production following delivery? Estrogen Oxytocin Progesterone Prolactin

Prolactin oxytocin - milk ejection

15-year-old girl presents with a recurrent, crampy lower abdominal pain, nausea, headache, and malaise. Her symptoms usually start 1 to 2 days before the onset of menstrual bleeding and then gradually diminishes over 12 to 72 hours. On exam, you note tenderness to palpation of the lower abdomen. Laboratory studies and pelvic ultrasound show no abnormal findings. Which of the following is the most likely cause of this patient's symptoms? Lymphoid hyperplasia Prostaglandin production Transmural colonic inflammation Twisting of the ovaries

Prostaglandin production primary dysmenorrhea

25-year-old woman who delivered her child 2 days ago is visited by a lactation consultant, who explains to her the importance of breastfeeding, especially before the body produces mature milk, due to the benefits of colostrum for the baby. Which of the following components is found in high quantities in colostrum? Carbohydrate Fat Protein Water

Protein

25-year-old G2P1 woman at 20 weeks gestation presents for a routine visit. Her blood pressure at this visit is 152/92 mm Hg. A repeat blood pressure taken the following day is 148/92 mm Hg. Which of the following additional findings is required for a diagnosis of preeclampsia? A third single systolic blood pressure reading ≥ 140 mm Hg Protein to creatinine ratio < 0.2 Proteinuria ≥ 300 mg in a 24-hour urine collection Thrombocytosis > 500,000/mcL

Proteinuria ≥ 300 mg in a 24-hour urine collection

A 25-year-old woman presents to the clinic for a routine wellness examination. She reports her husband is routinely controlling and humiliating her. Her husband has isolated her from her friends and family and manipulates her by withholding access to money. Which of the following is the most likely diagnosis? Physical intimate partner violence Post-traumatic stress disorder Psychological intimate partner violence Sexual intimate partner violence

Psychological intimate partner violence

19-year-old woman presents to your office due to sudden onset of malodorous yellowish-green vaginal discharge. She reports her symptoms began 7 days ago after having unprotected sexual intercourse with a new partner, who said he had no concerning symptoms. She reports 10 lifetime sexual partners. Her current medications include an oral contraceptive pill, which she takes reliably. Her past medical history includes bacterial vaginosis and gonorrhea. Upon further questioning, she reports she also has a 5-day history of dysuria and pain with intercourse. What physical exam finding will confirm your diagnosis? Cervical motion tenderness and thick, white vaginal discharge Flagellated protozoa moving in corkscrew motions Punctate macular hemorrhages on the cervix Thin, homogenous vaginal discharge with a fishy odor

Punctate macular hemorrhages on the cervix vaginitis - trichomoniasis

24-year-old woman at 14 weeks gestation reports to the clinic for her second prenatal visit. She reports intense nausea with occasional vomiting that has not resolved with dietary changes, including the addition of ginger and peppermint teas. Which of the following is the best next step in treatment? Chlorpromazine Doxylamine Droperidol Pyridoxine

Pyridoxine (vit B6) can add *doxylamine* if ineffective alone

21-year-old woman presents to her gynecologist with vaginal discharge for the past 2 days. She is sexually active with two male partners and occasionally uses condoms. A pelvic examination, Pap smear, wet prep, and nucleic acid amplification test are performed. Which of the following is associated with the most common bacterial cause of sexually transmitted infections? Change in microbiota with reduced concentration of lactobacilli Flesh-colored, cauliflower-like genital warts Growth on Thayer-Martin agar Pyuria with no organisms on Gram stain

Pyuria with no organisms on Gram stain chlamydia on urinalysis -- hard to gram stain

30-year-old multiparous woman presents with vaginal fullness and fecal incontinence. On exam, you note a bulge on the posterior vaginal wall that is worse with the Valsalva maneuver. Which of the following is the most likely diagnosis? Cystocele Endometriosis Rectocele Urethrocele

Rectocele

55-year-old woman with a history of a genital human papillomavirus-16 infection presents to the clinic with vaginal pruritus and irritation. Physical exam reveals right vulvar skin lesions. Which of the following is the most likely description of the vulvar lesions for the suspected diagnosis? Black nodules with irregular borders Black warty papules with a stuck-on appearance Clustered vesicles and well-demarcated erosions Red and white ulcerative lesions

Red and white ulcerative lesions vulvar cancer

55-year-old woman presents to the office due to vaginal fullness and pressure that has gotten worse over the past year. She feels a bulge at the opening of her vagina especially after being on her feet for several hours. She is also having more difficulty urinating. She does not have any problems with defecation and does not report vaginal bleeding or pain. She is widowed and has six adult children who were delivered vaginally. Which of the following is the most appropriate next step in treatment? Prescribe her a topical estrogen cream Prescribe her nitrofurantoin Refer her to a gynecologic surgeon for a surgical consult Refer her to a specialist for a pessary fitting

Refer her to a specialist for a pessary fitting initial uterine prolapse tx

35-year-old woman presents to her gynecologist for a follow-up visit to discuss her human papillomavirus (HPV) screening results. Her Pap test report came back positive for low-grade squamous intraepithelial lesion, and her reflex HPV test was negative. She has never had an abnormal HPV screen in the past. Which of the following is the best next step in management? Repeat cotesting in 12 months Repeat cotesting in 5 years Repeat human papillomavirus typing in 12 months Repeat Pap test in 12 months

Repeat cotesting in 12 months In patients ≥ 30 years of age with a Pap test result of low-grade squamous intraepithelial lesion and a negative reflex HPV test, repeat cotesting in 12 months is appropriate

25-year-old woman presents to her gynecologist with concerns about a mass in her left breast. She reports no family history of breast cancer. Physical examination reveals the presence of a single, well-defined, mobile, rubbery, nontender mass 2 cm in diameter in the right upper quadrant of the left breast. Ultrasound reveals the presence of a well-defined, solid mass. Which of the following is the most appropriate clinical intervention at this time? Cryoablation of the mass Repeat ultrasound and breast exam in 6 months Surgical excision of the mass Total mastectomy with lymph node dissection

Repeat ultrasound and breast exam in 6 months fibroadenoma biopsy-proven: cyroblation symptomatic, concerning: surgical excision

36-year-old G2P2 woman with no history of prenatal care for her current pregnancy had a spontaneous home vaginal delivery at 35 weeks gestation and presents with her baby 2 hours after delivery because she thinks he is sick. The infant is admitted to the neonatal intensive care unit with skin edema, ascites, pericardial effusion, pleural effusion, and a hemoglobin of 4.9 g/dL. The mother had no prenatal or postnatal care for her first pregnancy, which occurred 3 years ago. Her first child was delivered vaginally at home without any complications. The mother reports cocaine use during this pregnancy. Which of the following is the most likely contributing factor for the infant's condition? Advanced maternal age Cocaine use Preterm infant Rh incompatibility

Rh incompatibility second pregnancy without rhogam at 28 weeks and again after delivery Rh negative mother carrying Rh positive baby hemolytic disease of the newborn --> hydros fettles

26-year-old woman presents to the ED with sudden-onset pelvic pain, nausea, and vomiting for the past 2 hours. She reports no vaginal discharge or vaginal bleeding. Her medical history is significant for polycystic ovary syndrome. Her serum human chorionic gonadotropin is negative. Which of the following additional physical examination findings is most consistent with the suspected diagnosis? Boggy, tender, enlarged uterus Cervical motion tenderness Fixed, retroverted uterus Right-sided adnexal mass

Right-sided adnexal mass ovarian torsion, hx of ovarian cysts most likely cause torsion

27-year-old woman has delivered her first baby via vaginal delivery. The delivery happened quickly but smoothly, and a healthy baby boy is on her chest for skin-to-skin contact. After delivery of the placenta and uterine massage, careful examination of her vagina, perineum, and anorectum reveals tears of her vaginal epithelium and perineum, including the fascia of the perineal body. Her anal sphincter muscles do not appear to be involved. Which of the following is the best diagnosis of her injury? First-degree laceration Fourth-degree laceration Second-degree laceration Third-degree laceration

Second-degree laceration involve fascia and musculature of perineal body but do not affect the anal sphincter muscles

25-year-old woman develops lower abdominal pain, uterine tenderness, and fever on the second day postpartum after a cesarean delivery. She was started on an empiric antibiotic regimen of gentamicin and clindamycin but has a persistent fever after 3 days. Vital signs today are notable for T of 101.5°F, heart rate of 118 bpm, and blood pressure of 124/74 mm Hg. On exam, you notice focal uterine tenderness and a palpable cord-like mass on her lower right abdomen. The patient does not have a rash or demonstrate guarding or rebound tenderness. Which of the following is the most likely diagnosis? Appendicitis Peritonitis Septic pelvic thrombophlebitis Toxic shock syndrome

Septic pelvic thrombophlebitis complication of postpartum endometritis not resolving 3-5 days after antibiotics RF: C-section Sx: lower abdominal pain, fever, tachycardia, uterine tenderness, palpable cord-like mass

26-year-old woman presents to her primary care provider with increasing symptoms of anger, irritability, difficulty concentrating, feeling overwhelmed, and insomnia that have been causing significant interference to her work and daily activities. She has been recording the timing of her symptoms for the past 12 months and noticed her symptoms begin the week prior to her menses and resolve 2 to 3 days after menses. Her thyroid levels are within normal limits. Dysthymic disorder and major depressive disorder are ruled out. Which of the following medications is recommended as the first-line treatment for this patient? Alprazolam Leuprolide Sertraline Vitamin B6

Sertraline PMDD first line tx: SSRI

28-year-old G2P1001 woman at 26 weeks gestation arrives at the clinic for follow-up after a 50-gram, 1-hour oral glucose tolerance test, which showed a serum glucose of 150 mg/dL. Which of the following serum glucose levels confirms the suspected diagnosis after a 100-gram oral glucose load? Serum glucose of 130 mg/dL after 1 hour Serum glucose of 130 mg/dL after 3 hours Serum glucose of 165 mg/dL after 1 hour Serum glucose of 165 mg/dL after 2 hours

Serum glucose of 165 mg/dL after 2 hours positive one-step: 1 hour are > 130 mg/dL, if positive, continue to two step--> positive two-step OGTT: fasting glucose ≥ 95 mg/dL, 1-hour glucose ≥ 180 mg/dL, 2-hour glucose ≥ 155 mg/dL, and 3-hour glucose ≥ 140 mg/dL

25-year-old woman presents for follow-up 6 months postpartum because she has not menstruated since her vaginal delivery. She also mentions that she was not able to produce milk to breastfeed. You review her records from her labor and see that she had a postpartum hemorrhage resulting in hypotension that persisted for an hour. However, she did not have any instrumentation performed during the postpartum period. Her pregnancy test is negative. Which of the following diagnoses is this patient's presentation most concerning for? Asherman syndrome Hashimoto disease Pituitary adenoma Sheehan syndrome

Sheehan syndrome complication of postpartum hemorrhage (postpartum hypopituitarism - necrosis) signs: failure to lactate and amenorrhea or oligomenorrhea

24-year-old primigravid woman presents to the clinic with excessive nausea and vomiting for the past several weeks. Her last menstrual period was 2 months ago, and she states she had a positive home pregnancy test this week. Pelvic ultrasound reveals a 2 cm uterine mass with anechoic areas, increased blood flow, and myometrial invasion. No fetus is noted. Serum human chorionic gonadotropin level is 90,000 mIU/mL. Positron emission tomography reveals disease involving the uterus only. Which of the following is the appropriate clinical intervention for this patient? Hysterectomy Single-agent chemotherapy Uterine artery embolization Uterine wedge resection

Single-agent chemotherapy (methotrexate) gestational trophoblastic disease

65-year-old woman presents to her primary care provider with concerns about repeated fractures. She has had two wrist fractures and a hip fracture in the past 6 months. Since her hip fracture 2 months ago, she has been under a lot of stress and is only sleeping 5 hours at night. She has a 10 pack-year smoking history. Her current BMI is 30 kg/m2. She is not taking any medications or supplements. A dual-energy X-ray absorptiometry scan reveals a T-score of −2.8. Which of the following lifestyle modifications would be most beneficial for this patient? Improved sleep hygiene Smoking cessation Stress reduction Vitamin B12 supplementation

Smoking cessation osteoporosis

20-year-old woman presents to the office due to abdominal bloating and swelling in her legs and feet that coincides with her menstrual cycle. She starts having some abdominal and pelvic cramping around 1 week before her period, and it gets worse until her period starts. She states these symptoms have been getting worse over the past 6 months, and she has to wear bigger clothes and bigger shoes every month when she is on her period. Which of the following is the best regimen for the patient's symptoms? Acetazolamide Furosemide Spironolactone Tamoxifen

Spironolactone diuretic shown to improve symptoms of PMS (bloating, fluid retention, and breast tenderness)

40-year-old woman presents to a women's health clinic for a follow-up after an abnormal Pap smear and human papillomavirus typing that was positive for type 16. Erythematous, well-circumscribed, punctate cervical lesions are seen on colposcopy, and a biopsy is performed. Which of the following is the most common type of carcinoma associated with the underlying diagnosis? Adenocarcinoma Adenosquamous carcinoma Small cell carcinoma Squamous cell carcinoma

Squamous cell carcinoma MC type of Cervical carcinoma

30-year-old woman presents to her gynecologist for a colposcopy after an abnormal Pap smear. Examination of the cervix reveals dense, well-defined acetowhite areas near the squamocolumnar junction after acetic acid is applied. A biopsy of the dysplastic lesion is obtained. What is the most common type of carcinoma given the patient's most likely diagnosis? Adenocarcinoma Epithelial carcinoma Small cell carcinoma Squamous cell carcinoma

Squamous cell carcinoma MC type of cervical carcinoma Adeno (endometrial ca) epithelial (ovarian ca)

68-year-old woman presents to the clinic with a vulvar lesion. She reports the lesion is asymptomatic but has gradually grown over the past 6 weeks. Physical examination reveals a unifocal, firm, white plaque with some erosion. Biopsy confirms a malignant lesion. Which of the following is the most likely histologic type? Adenocarcinoma Basal cell carcinoma Epithelial cell cancer Squamous cell carcinoma

Squamous cell carcinoma (75% if vulvar cancer) Adeno - endometrial ca basal cell - skin ca epithelial - ovarian ca

62-year-old woman presents to her gynecologist with reports of vaginal heaviness and sensations of incomplete bladder emptying. A pelvic examination reveals uterine prolapse. Which stage of uterine prolapse is defined by the uterus located at the level of the hymen? Stage 1 Stage 2 Stage 3 Stage 4

Stage 2

72-year-old woman presents to the office with complaints of a new vaginal mass. While she was cleaning her house earlier in the day, she reports feeling a sudden fullness vaginally. She is worried her bladder is falling out. She is experiencing some discomfort that is worse when she is sitting. On physical exam, the external genitalia appear normal, and there is no frank blood. When the patient performs a Valsalva maneuver, the uterine cervix protrudes through the vaginal introitus by 5 mm. When relaxed, the prolapse is completely reduced. Bimanual exam reveals a 4 cm uterus with a smooth contour and no adnexal masses. There is anterior vaginal wall laxity. Which stage of pelvic organ prolapse is this patient experiencing? Stage 1 Stage 2 Stage 3 Stage 4

Stage 3 prolapse outside Introits but by no more than 1cm 1 - further than 1 cm from introitus 2 - 1cm or closer to introitus but does not protrude through with any Valsalva 3 - prolapse outside the vaginal introitus but by no more than 1 cm 4 - prolapse ≥ 1 cm outside the vaginal introitus

32-year-old woman who is breastfeeding her 12-week-old daughter presents to her primary care office reporting redness and tenderness to her right breast. She states her symptoms started yesterday afternoon and have progressed since then. On physical exam, there is redness, warmth, and swelling to her right nipple with a small amount of nipple discharge. Her left breast is normal. Which of the following is the most common cause of this patient's condition? Candida albicans Malassezia furfur Staphylococcus aureus Staphylococcus epidermidis

Staphylococcus aureus lactational mastitis

25-year-old G1P0 woman at 9 weeks gestation by last menstrual period presents to her obstetrician with nausea and vomiting. The patient states that she has vomited at least 5 times each day for the past 4 weeks and has lost about 9 pounds over that time period. Which of the following is most likely to be associated with the patient's condition? Anion gap metabolic acidosis Hyperchloremic alkalosis Hyperkalemia Starvation ketosis

Starvation ketosis hypOkalemia, hypOchloremic metabolic alkalosis secondary to vomiting, starvation ketosis if calorie intake is decreased, increased hematocrit secondary to hemoconcentration from plasma volume depletion, elevated blood urea nitrogen (BUN) and urine specific gravity secondary to hypovolemia, elevated liver enzymes, elevated amylase and lipase, mild hyperthyroidism, hypomagnesemia, and hypocalcemia Pyridoxine without hypovolemia: diphenhydramine, meclizine, or a dopamine antagonist (e.g., prochlorperazine, metoclopramide)

40-year-old woman presents to the clinic for her annual Pap test. On bimanual exam, right adnexal fullness is noted. Transvaginal ultrasound reveals a pedunculated mass near the right ovary that is attached to the uterus by a stalk of tissue. On surgical removal, this mass is determined to be a uterine leiomyoma. Which of the following classifications of leiomyoma is it? Cervical myoma Intramural myoma Submucosal myoma Subserosal myoma

Subserosal myoma originate from myometrium on serial surface -- pedunculate base, extend along broad ligament

A 24-year-old woman presents to the clinic complaining of mood instability. The symptoms are affecting her relationship with her boyfriend and her ability to perform at work. She specifically complains of moodiness, increased appetite, difficulty controlling her emotions, sleeping more than usual, difficulty concentrating, and bloating. Which of the following additional findings is required to fulfill the criteria for the most likely diagnosis? Absence of other psychiatric disease Symptoms occurring predominantly during the follicular phase Symptoms occurring predominantly during the luteal phase Symptoms starting during menses

Symptoms occurring predominantly during the luteal phase PMDD

A 26-year-old woman presents for routine prenatal care at 41 weeks gestation. You decide to induce labor. Which of the following is a known side effect of the medication typically used for labor induction? Headache Hypertension Pruritus Tachysystole

Tachysystole MC side effect of oxytocin to induce labor

A 42-year-old woman presents to the office due to increasing intermittent breast pain and tenderness that has gotten increasingly worse over the past year. She states the pain peaks about 1 week before her period. She has tried wearing more supportive bras and taking over-the-counter pain relievers for the past 6 months but nothing has helped. She feels like she is not able to work when she experiences this pain. She notices her breasts feel heavy and extremely tender, especially with physical activity. Physical exam reveals very dense breasts upon palpation with rope-like texture bilaterally. Which of the following therapy is the best next step for the treatment of her condition? Danazol Levonorgestrel Oxandrolone Tamoxifen

Tamoxifen fibrocystic breast changes danazol or tamoxifen (which preferred due to fewer side effects)

A 29-year-old G3P1102 pregnant woman presents to labor and delivery at 38 weeks and 0 days gestation complaining of leakage of fluid occurring 12 hours prior to arrival. Vital signs are T of 102.3°F, HR of 125 bpm, BP of 110/75 mm Hg, RR of 20/min, and oxygen saturation of 98% on room air. Physical examination reveals uterine tenderness and purulent drainage from the cervix with speculum exam. Laboratory studies show a white blood cell count of 14,000/μL and a C-reactive protein of 8.1 mg/L. Which of the following is one of the diagnostic criteria for the most likely diagnosis? C-reactive protein of at least 8.0 mg/L Temperature above 102.2°F Uterine tenderness White blood cell count of at least 14,000/μL

Temperature above 102.2°F Intra-amniotic infection (chorioamnionitis)

30-year-old woman presents to the clinic complaining of infertility. She says she has had painful menses and pain with intercourse for years. Transvaginal ultrasound shows a left-sided ovarian cystic mass with homogeneous low-medium-level echoes. Which of the following exam findings supports the most likely diagnosis? Bleeding from the cervical os Endocervical purulent discharge Tenderness in the posterior vaginal fornix Vaginal dryness and loss of rugae

Tenderness in the posterior vaginal fornix endometriosis (with endometrioma)

20-year-old woman presents to the clinic for treatment of newly diagnosed severe rheumatoid arthritis. She currently takes no medications, has no allergies, and her vital signs are within normal limits. Her last menstrual period was 3 weeks ago. She is in a monogamous relationship and does not use contraception. Which of the following is a side effect of methotrexate that makes it a poor initial treatment choice for this patient? Can cause a lethal rash Can cause severe anemia Can cause severe depression in younger patients Teratogenic and embryotoxic

Teratogenic and embryotoxic

24-year-old G1P0 woman at 12 weeks gestation by last menstrual period presents to her obstetrician for her initial prenatal visit. A routine nucleic acid amplification test is positive for Chlamydia trachomatis. The patient is treated with a single dose of azithromycin. Which of the following is the best next step in the management of this patient? Repeat testing for chlamydia and gonorrhea in 3 months Repeat testing for chlamydia only in 3 months Test of cure in 1 week Test of cure in 4 weeks

Test of cure in 4 weeks (in pregnant patients) additionally: repeat testing recommended 3 months after test of cure

A 47-year-old woman presents to the clinic due to irregular menstrual cycles for the past 6 months. She reports that the time between her menstrual cycles has increased. She also reports hot flashes and difficulty sleeping. Which of the following is the best way to confirm the suspected cause of her symptoms? Serum estradiol Serum follicle-stimulating hormone Serum luteinizing hormone The cause is best identified clinically

The cause is best identified clinically menopausal transition (during transition, hormone can fluctuate and be falsely normal)

29-year-old G1P0 woman at 11 weeks gestation presents to the ED with vaginal pain and bleeding. Transvaginal ultrasound reveals the presence of an intrauterine pregnancy. Fetal heart rate is 138 bpm. Her serum chorionic gonadotropin level was 39,488 mIU/mL 5 days ago and is currently 153,867 mIU/mL. Pelvic examination reveals the presence of a closed cervical os with a small amount of blood in the vaginal canal. What is the most likely diagnosis? Incomplete abortion Inevitable abortion Missed abortion Threatened abortion

Threatened abortion closed OS, increasing HcG, positive fetal heart rate

A 26-year-old woman presents to the office due to worsening depression, mood swings, anxiety, low energy, and poor concentration that begins about 2 weeks before her period and lasts until her period starts. She states she has had these symptoms since she was a teenager, but lately they have gotten so severe she has had to take many days off work and is currently at risk of losing her job. A physical examination is unremarkable. Which of the following diagnostic tests should be ordered in the workup of her condition? Adrenocorticotropic hormone stimulation test Fasting plasma glucose test Overnight sleep study Thyroid-stimulating hormone test

Thyroid-stimulating hormone test PMDD diagnosis by exclusion

25-year-old woman presents to the emergency department with a painful, swollen left breast and low-grade fever. She reports she developed mastitis while breastfeeding about 2 weeks ago that initially improved with antibiotics. She reports smoking one pack of cigarettes daily but always outside, away from the baby. She is taking no medications other than acetaminophen. She no longer has pain with breastfeeding but has developed a hot, painful, swollen area on her breast. On exam, she is obese but otherwise appears well. Her left breast has a localized, indurated area of about 3 cm diameter to the left of the areola, which is warm, tender to palpation, and fluctuant. Bedside ultrasound confirms the diagnosis and also facilitates aspiration of fluid. Which of the following elements of this patient's history puts her at greatest risk for this being a recurrent problem? Age Breastfeeding Obesity Tobacco use

Tobacco use associated with significant abscess recurrence

60-year-old woman presents to the clinic complaining of urinary incontinence. She says the symptoms primarily occur when she is laughing or sneezing. She has tried losing weight, reducing caffeine and alcohol intake, restricting water intake, and performing Kegel exercises without improvement. During pelvic examination, you notice vulvovaginal pallor, loss of vaginal rugae, and decreased elasticity. Which of the following is the best next step in management? Duloxetine Mirabegron Oxybutynin Topical estrogen

Topical estrogen overflow urinary incontinence

24-year-old woman presents to the emergency department with intense right lower quadrant pain for the past 12 hours. The patient is afebrile, tachycardic, and normotensive. Her last menstrual period was 8 weeks ago. She reports no vomiting or diarrhea but reports nausea and dyspareunia. Physical exam reveals right lower quadrant tenderness with guarding and without rebound tenderness. Vaginal exam reveals a closed cervical os and a right adnexal fullness on bimanual palpation. Which of the following represents the best next step in the diagnosis of this patient's pathology? Abdominal X-ray Computed tomography of abdomen and pelvis without contrast Magnetic resonance imaging of abdomen and pelvis without contrast Transvaginal pelvic ultrasound

Transvaginal pelvic ultrasound ectopic pregnancy

A 32-year-old premenopausal woman with no significant medical history presents to the clinic due to a heavy menstrual flow that lasts 12 days on average. This bleeding is not predictable in onset and causes her severe social and physical distress. Urine human chorionic gonadotropin testing is negative. Which of the following is the best initial diagnostic study? Computed tomography Hysteroscopy Magnetic resonance imaging Transvaginal ultrasonography

Transvaginal ultrasonography

26-year-old woman presents to the emergency department with complaints of lower abdominal pain and vaginal discharge for the past 5 days. She also reports chills and malaise. Vital signs are T of 100.3°F, HR of 105 bpm, BP of 145/94 mm Hg, and RR of 20/min. On physical exam, she has tenderness to palpation in the left lower quadrant without guarding or rebound. Pelvic exam reveals purulent discharge from the cervical os, cervical motion tenderness, and left-sided adnexal tenderness. Transvaginal ultrasound shows an inflammatory multilocular mass in the left adnexal area. Which of the following is the most likely diagnosis? Acute diverticulitis Ruptured ovarian cyst Ruptured tubo-ovarian abscess Tubo-ovarian abscess without rupture

Tubo-ovarian abscess without rupture Presentation similar to PID (which is also a RF) *Ovarian cyst will show cyst surrounded by free fluid on US *ruptured TOA will have signs of acute abdomen (guarding and rebound tenderness)

27-year-old G1P0 woman at 12 weeks gestation presents to her obstetrician for a routine prenatal visit. Ultrasound confirms the presence of a twin pregnancy. Which of the following is the most likely complication of multiple gestation? Large for gestational age infants Maternal breast cancer Post-term delivery Twin-twin transfusion syndrome

Twin-twin transfusion syndrome

23-year-old G1P0 woman at 37 weeks gestation presents to the labor and delivery unit for an uncomplicated vaginal delivery. Her cervix is 8 cm dilated and 100% effaced. A fetal heart rate monitor is attached, and the baseline fetal heart rate is 140 bpm. Which of the following fetal heart rates is considered reactive? One acceleration of 15 bpm above baseline for 2 minutes in a 20-minute period One acceleration of 20 bpm above baseline for 20 seconds in a 20-minute period Two accelerations of 10 bpm above baseline for 10 seconds each in a 10-minute period Two accelerations of 15 bpm above baseline for 15 seconds each in a 20-minute period

Two accelerations of 15 bpm above baseline for 15 seconds each in a 20-minute period

34-year-old woman who had an uncomplicated vaginal delivery about 1 week ago presents to the office with brownish-red, watery vaginal discharge. She is afebrile, and there is no tenderness to palpation to the abdomen and pelvis. The uterus is also nonpalpable. The vaginal vault is pink and slightly atrophic, and there is a moderate amount of brownish-red discharge present. There is no bright red blood or evidence of trauma. Which of the following is the best diagnosis of this condition? Endometritis Perineal laceration Postpartum hemorrhage Typical discharge of lochia serosa

Typical discharge of lochia serosa brownish-red and watery discharge

28-year-old woman presents to the clinic due to mastalgia for the past 2 months. She reports her pain increases with the onset of her menstrual cycle. Physical exam demonstrates a smooth, firm, discrete mobile mass. Which of the following diagnostic studies is best indicated at this time? Core-needle biopsy Diagnostic mammography Fine-needle aspiration Ultrasonography

Ultrasonography fibrocystic disease initial eval if rapid changes develop - need biopsy

55-year-old postmenopausal woman presents to the clinic with vaginal spotting. She says she has not had any vaginal bleeding since her last period 4 years ago. Which of the following is a risk factor for the most concerning diagnosis? Combined hormonal contraception use Late menarche Smoking cigarettes Unopposed estrogen therapy

Unopposed estrogen therapy uterine endometrial cancer RF

37-year-old woman presents to the office to ask about female contraceptive barrier methods and wants to be fitted for a cervical cap or diaphragm. Which of the following is a side effect of using these devices? Menorrhagia Pelvic inflammatory disease Urinary tract infection Urogenital fistula formation

Urinary tract infection

32-year-old G3P2 woman with no significant medical history is 3 hours postpartum after giving birth to her full-term baby girl. There were no complications during her delivery, and the placenta was delivered intact about 12 minutes after the baby. She is now noted to be hypotensive with a blood pressure of 80/50 mm Hg and tachycardic with a heart rate of 135 bpm. Which of the following is the most common cause of postpartum hemorrhage? Maternal coagulopathy Perineal laceration Retained placental tissue Uterine atony

Uterine atony

27-year-old G3P2002 presents to the labor and delivery unit at 32 6/7 weeks gestational age with lower abdominal cramping. She reports the cramping pain has been increasing in frequency throughout the day but reports no vaginal bleeding or leakage of fluids. A fetal fibronectin test is performed and is negative. Which of the following findings would meet the diagnostic criteria for preterm labor? Uterine contractions 1 minute apart and a cervical dilation of 1 cm Uterine contractions 1 minute apart and a cervical length of 30 mm on transvaginal ultrasound Uterine contractions 5 minutes apart and a cervical dilation of 3 cm Uterine contractions 5 minutes apart and a cervical length of 30 mm on transvaginal ultrasound

Uterine contractions 5 minutes apart and a cervical dilation of 3 cm contractions 5 minutes apart with cervical changes (dilation >3cm or cervical length <20mm (or 20-30 with positive fetal fibronectin))

62-year-old postmenopausal woman is diagnosed with metastatic hormone receptor-positive, human epidermal growth factor receptor 2-negative breast cancer, for which she is prescribed a single-agent therapeutic treatment. Which of the following is considered a black box warning associated with this medication? Cardiomyopathy Decreased bone mineral density Pulmonary toxicity Uterine malignancy

Uterine malignancy Breast cancer Postmenopausal: single agent endocrine therapy -- tamoxifen (causes *antiestrogenenic effects* > endometrial hyperplasia and uterine malignancy)

A woman who delivered her child vaginally 2 hours ago begins to report a "racing heart," palpitations, and shortness of breath. Blood loss during delivery was estimated at 600 mL, and the patient has had ongoing bleeding since delivery. Initial stabilization included intravenous fluids, tranexamic acid, and an increase in the rate of oxytocin administration. Which of the following procedures is the first-line intervention for the most common cause of this patient's condition after initial stabilization? Arterial embolization B-lynch compression sutures Hysterectomy Uterine massage

Uterine massage Postpartum Hemorrhage (PPH) MCC uterine atony

52-year-old woman presents to the gynecology clinic with vaginal dryness. She also reports vaginal burning, dyspareunia, and urinary frequency. The patient reports no hot flashes. Pelvic examination reveals vulvovaginal erythema and fissuring. Which of the following is the initial first-line treatment for the most likely diagnosis? Systemic estrogen and progestin Vaginal dehydroepiandrosterone Vaginal estrogen Vaginal moisturizers and lubricants

Vaginal moisturizers and lubricants initial line vulvovaginal atrophy due to decline serum estrogen

21-year-old G1P0 woman at 8 weeks gestation presents to her obstetrician for her first prenatal visit. She has been taking multiple over-the-counter vitamin supplements. Which of the following vitamins, if taken in excess, is teratogenic in the first trimester and is associated with spontaneous abortion and fetal malformation? Vitamin A Vitamin C Vitamin D Vitamin E

Vitamin A

44-year-old woman with uncontrolled diabetes mellitus presents to the gynecologic clinic with vulvar pruritus, vaginal burning, vaginal irritation, and dysuria. Physical examination reveals vulvar erythema. Urinalysis has no leukocyte esterase or nitrites. The vaginal pH is 4.25. Which of the following is the most common finding in the most likely condition? Dysuria Vaginal burning Vaginal irritation Vulvar pruritus

Vulvar pruritus vulvovaginal candidiasis pH <4.5

22-year-old woman presents to her primary care provider with recurrent bilateral breast pain each month. Physical exam reveals no masses, skin changes, nipple discharge, or axillary lymphadenopathy. Which part of her menstrual cycle is the most likely timing for this pain to occur in? During menses First week after menses Week before menses Week of ovulation

Week before menses fibrocystic changes

41-year-old nulliparous woman with a history of infertility presents to her gynecologist with nausea, vomiting, and pelvic discomfort. Her last menstrual period was 7 weeks ago. A serum human chorionic gonadotropin level is obtained and is 346,000 mIU/mL. A transvaginal ultrasound reveals the absence of amniotic fluid and a snowstorm pattern. The patient is scheduled for surgical treatment. Which of the following is the most appropriate follow-up for the patient after surgery? Monthly monitoring of serum human chorionic gonadotropin until < 5 mIU/mL Monthly ultrasounds for 3 months Weekly monitoring of serum human chorionic gonadotropin until < 5 mIU/mL Weekly ultrasounds for 4 weeks

Weekly monitoring of serum human chorionic gonadotropin until < 5 mIU/mL hydatidiform mole

A 24-year-old woman presents to the office for evaluation of infertility after trying to conceive for several years. A transabdominal ultrasound reveals a "string of pearls" appearance to the bilateral ovaries. Which of the following clinical findings will she most likely have on a history and physical examination? Dull facial expression, unexplained weight gain, bradycardia, and dry skin Excessive sweating, weight gain, abdominal striae, and a buffalo hump Increasing fatigue, muscle weakness, darkening skin, and hypotension Weight gain, hirsutism, acne, and insulin resistance

Weight gain, hirsutism, acne, and insulin resistance Polycystic ovarian syndrome (PCOS)


Related study sets

2.5 Case Q→Q: Correlation Coefficient-r

View Set

Unit 32: Pediatric Musculoskeletal, Neuromuscular, Neurocognitive Disorders/Cognitive Impairment

View Set

Financial Accounting Exam 2 Chapters 4-6

View Set

Intro to Business Chapter Thirteen Quiz

View Set

VN 131 STUDY GUIDE CH. 16,17,18 CARDIO

View Set

UNIT 5: other investment vehicles

View Set

RENAISSANCE GOVERNMENT: FRANCE AND ENGLAND

View Set

Lecture 2 : Diagnostic Testing : Sensitivity and Specificity

View Set